Docsity
Docsity

Prepare for your exams
Prepare for your exams

Study with the several resources on Docsity


Earn points to download
Earn points to download

Earn points by helping other students or get them with a premium plan


Guidelines and tips
Guidelines and tips

NCLEX NCLEX-RN MULTIPLE QUESTIONS WITH CORECT ANSWERS LATEST UPDATED EXAMINATION STUDY GUI, Exams of Nursing

NCLEX NCLEX-RN MULTIPLE QUESTIONS WITH CORECT ANSWERS LATEST UPDATED EXAMINATION STUDY GUIDE 2023

Typology: Exams

2022/2023

Available from 02/02/2023

charleswest
charleswest 🇺🇸

4

(10)

470 documents

1 / 333

Toggle sidebar

Related documents


Partial preview of the text

Download NCLEX NCLEX-RN MULTIPLE QUESTIONS WITH CORECT ANSWERS LATEST UPDATED EXAMINATION STUDY GUI and more Exams Nursing in PDF only on Docsity! NCLEX NCLEX-RN MULTIPLE QUESTIONS WITH CORECT ANSWERS LATEST UPDATED EXAMINATION STUDY GUIDE 2023 QUESTION 1 Which classification of drugs is contraindicated for the client with hypertrophic cardiomyopathy? A. Positive inotropes B. Vasodilators C. Diuretics D. Antidysrhythmics Answer: A Explanation: (A) Positive inotropic agents should not be administered owing to their action of increasing myocardial contractility. Increased ventricular contractility would increase outflow tract obstruction in the client with hypertrophic cardiomyopathy. (B) Vasodilators are not typically prescribed but are not contraindicated. (C) Diuretics are used with caution to avoid causing hypovolemi A. (D) Antidysrhythmics are typically needed to treat both atrial and ventricular dysrhythmias. QUESTION 2 Signs and symptoms of an allergy attack include which of the following? A. Wheezing on inspiration B. Increased respiratory rate C. Circumoral cyanosis D. Prolonged expiration Answer: D Explanation: (A) Wheezing occurs during expiration when air movement is impaired because of constricted edematous bronchial lumin A. (B) Respirations are difficult, but the rate is frequently normal. (C) The circumoral area is usually pale. Cyanosis is not an early sign of hypoxi A. (D) Expiration is prolonged because the alveoli are greatly distended and air trapping occurs. QUESTION 3 A client confides to the nurse that he tasted poison in his evening meal. This would be an example of what type of hallucination? A. Auditory B. Gustatory NCLEX NCLEX-RN MULTIPLE QUESTIONS WITH CORECT ANSWERS LATEST UPDATED EXAMINATION STUDY GUIDE 2023 C. Olfactory D. Visceral Answer: B Explanation: (A) Auditory hallucinations involve sensory perceptions of hearing. (B) Gustatory hallucinations involve sensory perceptions of taste. (C) Olfactory hallucinations involve sensory perceptions of smell. (D) Visceral hallucinations involve sensory perceptions of sensation. NCLEX NCLEX-RN MULTIPLE QUESTIONS WITH CORECT ANSWERS LATEST UPDATED EXAMINATION STUDY GUIDE 2023 finger foods allow him to eat during periods of activity. (B) This type of therapy should be implemented when other methods have been exhausted. (C) The manic client should not be in control of his treatment plan. This type of client may forget to eat. (D) The manic client is unable to sit down to eat full meals. QUESTION 7 A client with bipolar disorder taking lithium tells the nurse that he has ringing in his ears, blurred vision, and diarrhe A. The nurse notices a slight tremor in his left hand and a slurring pattern to his speech. Which of the following actions by the nurse is appropriate? A. Administer a stat dose of lithium as necessary. B. Recognize this as an expected response to lithium. C. Request an order for a stat blood lithium level. D. Give an oral dose of lithium antidote. Answer: C Explanation: (A) These symptoms are indicative of lithium toxicity. A stat dose of lithium could be fatal. (B) These are toxic effects of lithium therapy. (C) The client is exhibiting symptoms of lithium toxicity, which may be validated by lab studies. (D) There is no known lithium antidote. QUESTION 8 A diagnosis of hepatitis C is confirmed by a male client’s physician. The nurse should be knowledgeable of the differences between hepatitis A, B, and C. Which of the following are characteristics of hepatitis C? A. The potential for chronic liver disease is minimal. B. The onset of symptoms is abrupt. C. The incubation period is 2–26 weeks. D. There is an effective vaccine for hepatitis B, but not for hepatitis C. Answer: C Explanation: (A) Hepatitis C and B may result in chronic liver disease. Hepatitis A has a low potential for chronic liver disease. (B) Hepatitis C and B have insidious onsets. Hepatitis A has an abrupt onset. (C) Incubation periods are as follows: hepatitis C is 2–26 weeks, hepatitis B is 6–20 weeks, and hepatitis A is 2–6 weeks. (D) Only hepatitis B has an effective vaccine. QUESTION 9 Hypoxia is the primary problem related to near-drowning victims. The first organ that sustains irreversible damage after submersion in water is the: A. Kidney (urinary system) NCLEX NCLEX-RN MULTIPLE QUESTIONS WITH CORECT ANSWERS LATEST UPDATED EXAMINATION STUDY GUIDE 2023 B. Brain (nervous system) C. Heart (circulatory system) D. Lungs (respiratory system) Answer: B Explanation: NCLEX NCLEX-RN MULTIPLE QUESTIONS WITH CORECT ANSWERS LATEST UPDATED EXAMINATION STUDY GUIDE 2023 (A) The kidney can survive after 30 minutes of water submersion. (B) The cerebral neurons sustain irreversible damage after 4–6 minutes of water submersion. (C) The heart can survive up to 30 minutes of water submersion. (D) The lungs can survive up to 30 minutes of water submersion. QUESTION 10 Which of the following activities would be most appropriate during occupational therapy for a client with bipolar disorder? A. Playing cards with other clients B. Working crossword puzzles C. Playing tennis with a staff member D. Sewing beads on a leather belt Answer: C Explanation: (A) This activity is too competitive, and the manic client might become abusive toward the other clients. (B) During mania, the client’s attention span is too short to accomplish this task. (C) This activity uses gross motor skills, eases tension, and expands excess energy. A staff member is better equipped to interact therapeutically with clients. (D) This activity requires the use of fine motor skills and is very tedious. QUESTION 11 A 30-year-old male client is admitted to the psychiatric unit with a diagnosis of bipolar disorder. For the last 2 months, his family describes him as being “on the move,” sleeping 3–4 hours nightly, spending lots of money, and losing approximately 10 lb. During the initial assessment with the client, the nurse would expect him to exhibit which of the following? A. Short, polite responses to interview questions B. Introspection related to his present situation C. Exaggerated self-importance D. Feelings of helplessness and hopelessness Answer: C Explanation: (A) During the manic phase of bipolar disorder, clients have short attention spans and may be abusive toward authority figures. (B) Introspection requires focusing and concentration; clients with mania experience flight of ideas, which prevents concentration. (C) Grandiosity and an inflated sense of self-worth are characteristic of this disorder. (D) Feelings of helplessness and hopelessness are symptoms of the depressive stage of bipolar disorder. QUESTION 12 Diabetes during pregnancy requires tight metabolic control of glucose levels to prevent perinatal mortality. When evaluating the pregnant client, the nurse knows the recommended serum glucose range during pregnancy NCLEX NCLEX-RN MULTIPLE QUESTIONS WITH CORECT ANSWERS LATEST UPDATED EXAMINATION STUDY GUIDE 2023 risk. (D) There is no evidence that any religious groups are at higher risk. QUESTION 15 A schizophrenic client has made sexual overtures toward her physician on numerous occasions. During lunch, the client tells the nurse, “My doctor is in love with me and wants to marry me.” This client is using which of the following defense mechanisms? NCLEX NCLEX-RN MULTIPLE QUESTIONS WITH CORECT ANSWERS LATEST UPDATED EXAMINATION STUDY GUIDE 2023 A. Displacement B. Projection C. Reaction formation D. Suppression Answer: B Explanation: (A) Displacement involves transferring feelings to a more acceptable object. (B) Projection involves attributing one’s thoughts or feelings to another person. (C) Reaction formation involves transforming an unacceptable impulse into the opposite behavior. (D) Suppression involves the intentional exclusion of unpleasant thoughts or experiences. QUESTION 16 When teaching a sex education class, the nurse identifies the most common STDs in the United States as: A. Chlamydia B. Herpes genitalis C. Syphilis D. Gonorrhea Answer: A Explanation: (A) Chlamydia trachomatis infection is the most common STD in the United States. The Centers for Disease Control and Prevention recommend screening of all high-risk women, such as adolescents and women with multiple sex partners. (B) Herpes simplex genitalia is estimated to be found in 5–20 million people in the United States and is rising in occurrence yearly. (C) Syphilis is a chronic infection caused by Treponema pallidum. Over the last several years the number of people infected has begun to increase. (D) Gonorrhea is a bacterial infection caused by the organism Neisseria gonorrhoeae. Although gonorrhea is common, chlamydia is still the most common STD. QUESTION 17 The nurse assists a client with advanced emphysema to the bathroom. The client becomes extremely short of breath while returning to bed. The nurse should: A. Increase his nasal O2 to 6 L/min B. Place him in a lateral Sims’ position C. Encourage pursed-lip breathing D. Have him breathe into a paper bag Answer: C Explanation: NCLEX NCLEX-RN MULTIPLE QUESTIONS WITH CORECT ANSWERS LATEST UPDATED EXAMINATION STUDY GUIDE 2023 (A) Giving too high a concentration of O2 to a client with em-physema may remove his stimulus to breathe. (B) The client should sit forward with his hands on his knees or an overbed table and with shoulders elevated. (C) Pursed-lip breathing helps the client to blow off CO2 and to keep air passages open. (D) Covering the face of a client extremely short of breath may cause anxiety and further increase dyspnea. NCLEX NCLEX-RN MULTIPLE QUESTIONS WITH CORECT ANSWERS LATEST UPDATED EXAMINATION STUDY GUIDE 2023 QUESTION 21 An 8-year-old child comes to the physician’s office complaining of swelling and pain in the knees. His mother says, “The swelling occurred for no reason, and it keeps getting worse.” The initial diagnosis is Lyme disease. When talking to the mother and child, questions related to which of the following would be important to include in the initial history? A. A decreased urinary output and flank pain B. A fever of over 103F occurring over the last 2–3 weeks C. Rashes covering the palms of the hands and the soles of the feet D. Headaches, malaise, or sore throat Answer: D Explanation: (A) Urinary tract symptoms are not commonly associated with Lyme disease. (B) A fever of 103F is not characteristic of Lyme disease. (C) The rash that is associated with Lyme diseasedoes not appear on the palms of the hands and the soles of the feet. (D) Classic symptoms of Lyme disease include headache, malaise, fatigue, anorexia, stiff neck, generalized lymphadenopathy, splenomegaly, conjunctivitis, sore throat, abdominal pain, and cough. QUESTION 22 When administering phenytoin (Dilantin) to a child, the nurse should be aware that a toxic effect of phenytoin therapy is: A. Stephens-Johnson syndrome B. Folate deficiency C. Leukopenic aplastic anemia D. Granulocytosis and nephrosis Answer: A Explanation: (A) Stephens-Johnson syndrome is a toxic effect of phenytoin. (B) Folate deficiency is a side effect of phenytoin, but not a toxic effect. (C) Leukopenic aplastic anemia is a toxic effect of carbamazepine (Tegretol). (D) Granulocytosis and nephrosis are toxic effects of trimethadione (Tridione). QUESTION 23 The nurse should know that according to current thinking, the most important prognostic factor for a client with breast cancer is: A. Tumor size B. Axillary node status C. Client’s previous history of disease D. Client’s level of estrogen-progesterone receptor assays NCLEX NCLEX-RN MULTIPLE QUESTIONS WITH CORECT ANSWERS LATEST UPDATED EXAMINATION STUDY GUIDE 2023 Answer: B Explanation: (A) Although tumor size is a factor in classification of cancer growth, it is not an indicator of lymph node spread. (B) Axillary node status is the most important indicator for predicting how far the cancer has spread. If the lymph nodes are positive for cancer cells, the prognosis is poorer. (C) The client’s previous history of cancer NCLEX NCLEX-RN MULTIPLE QUESTIONS WITH CORECT ANSWERS LATEST UPDATED EXAMINATION STUDY GUIDE 2023 puts her at an increased risk for breast cancer recurrence, especially if the cancer occurred in the other breast. It does not predict prognosis, however. (D) The estrogen-progesterone assay test is used to identify present tumors being fedfrom an estrogen site within the body. Some breast cancers grow rapidly as long as there is an estrogen supply such as from the ovaries. The estrogen-progesterone assay test does not indicate the prognosis. QUESTION 24 Three weeks following discharge, a male client is readmitted to the psychiatric unit for depression. His wife stated that he had threatened to kill himself with a handgun. As the nurse admits him to the unit, he says, “I wish I were dead because I am worthless to everyone; I guess I am just no good.” Which response by the nurse is most appropriate at this time? A. “I don’t think you are worthless. I’m glad to see you, and we will help you.” B. “Don’t you think this is a sign of your illness?” C. “I know with your wife and new baby that you do have a lot to live for.” D. “You’ve been feeling sad and alone for some time now?” Answer: D Explanation: (A) This response does not acknowledge the client’s feelings. (B) This is a closed question and does not encourage communication. (C) This response negates the client’s feelings and does not require a response from the client. (D) This acknowledges the client’s implied thoughts and feelings and encourages a response. QUESTION 25 Which of the following should be included in discharge teaching for a client with hepatitis C? A. He should take aspirin as needed for muscle and joint pain. B. He may become a blood donor when his liver enzymes return to normal. C. He should avoid alcoholic beverages during his recovery period. D. He should use disposable dishes for eating and drinking. Answer: C Explanation: (A) Aspirin is hepatotoxic, may increase bleeding, and should be avoided. (B) Blood should not be donated by a client who has had hepatitis C because of the possibility of transmission of disease. (C) Alcohol is detoxified in the liver. (D) Hepatitis C is not spread through the oral route. QUESTION 26 The initial treatment for a client with a liquid chemical burn injury is to: A. Irrigate the area with neutralizing solutions B. Flush the exposed area with large amounts of water NCLEX NCLEX-RN MULTIPLE QUESTIONS WITH CORECT ANSWERS LATEST UPDATED EXAMINATION STUDY GUIDE 2023 Lyme disease? A. Complete bed rest for 6–8 weeks B. Tetracycline treatment C. IV amphotericin B D. High-protein diet with limited fluids NCLEX NCLEX-RN MULTIPLE QUESTIONS WITH CORECT ANSWERS LATEST UPDATED EXAMINATION STUDY GUIDE 2023 Answer: B Explanation: (A) The client is not placed on complete bed rest for 6 weeks. (B) Tetracycline is the treatment of choice for children with Lyme disease who are over the age of 9. (C) IV amphotericin B is the treatment for histoplasmosis. (D) The client is not restricted to a high-protein diet with limited fluids. QUESTION 30 The physician recommends immediate hospital admission for a client with PIH. She says to the nurse, “It’s not so easy for me to just go right to the hospital like that.” After acknowledging her feelings, which of these approaches by the nurse would probably be best? A. Stress to the client that her husband would want her to do what is best for her health. B. Explore with the client her perceptions of why she is unable to go to the hospital. C. Repeat the physician’s reasons for advising immediate hospitalization. D. Explain to the client that she is ultimately responsible for her own welfare and that of her baby. Answer: B Explanation: (A) This answer does not hold the client accountable for her own health. (B) The nurse should explore potential reasons for the client’s anxiety: are there small children at home, is the husband out of town? The nurse should aid the client in seeking support or interventions to decrease the anxiety of hospitalization. (C) Repeating the physician’s reason for recommending hospitalization may not aid the client in dealing with her reasons for anxiety. (D) The concern for self and welfare of baby may be secondary to a woman who is in a crisis situation. The nurse should explore the client’s potential reasons for anxiety. For example, is there another child in the home who is ill, or is there a husband who is overseas and not able to return on short notice? QUESTION 31 The child with iron poisoning is given IV deforoxamine mesylate (Desferal). Following administration, the child suffers hypotension, facial flushing, and urticari A. The initial nursing intervention would be to: A. Discontinue the IV B. Stop the medication, and begin a normal saline infusion C. Take all vital signs, and report to the physician D. Assess urinary output, and if it is 30 mL an hour, maintain current treatment Answer: B Explanation: (A) The IV line should not be discontinued because other IV medications will be needed. (B) Stop the medication and begin a normal saline infusion. The child is exhibiting signs of an allergic reaction NCLEX NCLEX-RN MULTIPLE QUESTIONS WITH CORECT ANSWERS LATEST UPDATED EXAMINATION STUDY GUIDE 2023 and could go into shock if the medication is not stopped. The line should be kept opened for other medication. (C) Taking vital signs and reporting to the physician is not an adequate intervention because the IV medication continues to flow. (D) Assessing urinary output and, if it is 30 mL an hour, maintaining current treatment is an inappropriate intervention owing to the child’s obvious allergic reaction. NCLEX NCLEX-RN MULTIPLE QUESTIONS WITH CORECT ANSWERS LATEST UPDATED EXAMINATION STUDY GUIDE 2023 Answer: A Explanation: (A) Sinus bradycardia and atrioventricular (AV) heart block are usually a result of right coronary artery occlusion. The right coronary artery perfuses the sinoatrial and AV nodes in mostindividuals. (B) Occlusion of the left main coronary artery causes bundle branch blocks and premature ventricular contractions. (C) Occlusion of the circumflex artery does not cause bradycardi A. (D) Sinus tachycardia occurs primarily with left anterior descending coronary artery occlusion because this form of occlusion impairs left ventricular function. QUESTION 35 When the nurse is evaluating lab data for a client 18–24 hours after a major thermal burn, the expected physiological changes would include which of the following? A. Elevated serum sodium B. Elevated serum calcium C. Elevated serum protein D. Elevated hematocrit Answer: D Explanation: (A) Sodium enters the edema fluid in the burned area, lowering the sodium content of the vascular fluid. Hyponatremia may continue for days to several weeks because of sodium loss to edema, sodium shifting into the cells, and later, diuresis. (B) Hypocalcemia occurs because of calcium loss to edema fluid at the burned site (third space fluid). (C) Protein loss occurs at the burn site owing to increased capillary permeability. Serum protein levels remain low until healing occurs. (D) Hematocrit level is elevated owing to hemoconcentration from hypovolemi A. Anemia is present in the postburn stage owing to blood loss and hemolysis, but it cannot be assessed until the client is adequately hydrated. QUESTION 36 What is the most effective method to identify early breast cancer lumps? A. Mammograms every 3 years B. Yearly checkups performed by physician C. Ultrasounds every 3 years D. Monthly breast self-examination Answer: D Explanation: (A) Mammograms are less effective than breast self-examination for the diagnosis of abnormalities in younger women, who have denser breast tissue. They are more effective forwomen older than 40. (B) Up to 15% of early-stage breast cancers are detected by physical examination; however, 95% are detected by women doing NCLEX NCLEX-RN MULTIPLE QUESTIONS WITH CORECT ANSWERS LATEST UPDATED EXAMINATION STUDY GUIDE 2023 breast self-examination. (C) Ultrasound is used primarily to determine the location of cysts and to distinguish cysts from solid masses. (D) Monthly breast self-examination has been shown to be the most effective method for early detection of breast cancer. Approximately 95% of lumps are detected by women themselves. QUESTION 37 A client with a C-3–4 fracture has just arrived in the emergency room. The primary nursing intervention is: NCLEX NCLEX-RN MULTIPLE QUESTIONS WITH CORECT ANSWERS LATEST UPDATED EXAMINATION STUDY GUIDE 2023 A. Stabilization of the cervical spine B. Airway assessment and stabilization C. Confirmation of spinal cord injury D. Normalization of intravascular volume Answer: B Explanation: (A) If cervical spine injury is suspected, the airway should be maintained using the jaw thrust method that also protects the cervical spine. (B) Primary intervention is protection of the airway and adequate ventilation. (C, D) All other interventions are secondary to adequate ventilation. QUESTION 38 To ensure proper client education, the nurse should teach the client taking SL nitroglycerin to expect which of the following responses with administration? A. Stinging, burning when placed under the tongue B. Temporary blurring of vision C. Generalized urticaria with prolonged use D. Urinary frequency Answer: A Explanation: (A) Stinging or burning when nitroglycerin is placed under the tongue is to be expected. This effect indicates that the medication is potent and effective for use. Failure to have this response means that the client needs to get a new bottle of nitroglycerin. (B, C, D) The other responses are not expected in this situation and are not even side effects. QUESTION 39 Nursing care for the substance abuse client experiencing alcohol withdrawal delirium includes: A. Maintaining seizure precautions B. Restricting fluid intake C. Increasing sensory stimuli D. Applying ankle and wrist restraints Answer: A Explanation: (A) These clients are at high risk for seizures during the 1st week after cessation of alcohol intake. (B) Fluid intake should be increased to prevent dehydration. (C) Environmental stimuli should be decreased to prevent precipitation of seizures. (D) Application of restraints may cause the client to increase his or her physical activity and may eventually lead to exhaustion. NCLEX NCLEX-RN QUESTION 46 A client returns for her 6-month prenatal checkup and has gained 10 lb in 2 months. The results of her physical A. Determine gross abnormal motor function B. Obtain a baseline for comparison with the infant’s future adaptation to the environment C. Evaluate the infant’s vital functions D. Determine the extent of congenital malformations Answer: C Explanation: (A) Apgar scores are not related to the infant’s care, but to the infant’s physical condition. (B) Apgar scores assess the current physical condition of the infant and are not related to future environmental adaptation. (C) The purpose of the Apgar system is to evaluate the physical condition of the newborn at birth and to determine if there is an immediate need for resuscitation. (D) Congenital malformations are not one of the areas assessed with Apgar scores. QUESTION 44 Hematotympanum and otorrhea are associated with which of the following head injuries? A. Basilar skull fracture B. Subdural hematoma C. Epidural hematoma D. Frontal lobe fracture Answer: A Explanation: (A) Basilar skull fractures are fractures of the base of the skull. Blood behind the eardrum or blood or cerebrospinal fluid (CSF) leaking from the ear are indicative of a dural laceration. Basilar skull fractures are the only type with these symptoms. (B, C, D) These do not typically cause dural lacerations and CSF leakage. QUESTION 45 Which of the following risk factors associated with breast cancer would a nurse consider most significant in a client’s history? A. Menarche after age 13 B. Nulliparity C. Maternal family history of breast cancer D. Early menopause Answer: C Explanation: (A) Women who begin menarche late (after 13 years old) have a lower risk of developing breast cancer than women who have begun earlier. Average age for menarche is 12.5 years. (B) Women who have never been pregnant have an increased risk for breast cancer, but a positive family history poses an even greater risk. (C) A positive family history puts a woman at an increased risk of developing breast cancer. It is recommended that mammography screening begin 5 years before the age at which an immediate female relative was diagnosed with breast cancer. (D) Early menopause decreases the risk of developing breast cancer. NCLEX NCLEX-RN decreased peripheral perfusion and bradycardi A. (B) Dobutamine’s side effects include increased heart rate and blood pressure, ventricular ectopy, nausea, examination are normal. How does the nurse interpret the effectiveness of the instruction about diet and weight control? A. She is compliant with her diet as previously taught. B. She needs further instruction and reinforcement. C. She needs to increase her caloric intake. D. She needs to be placed on a restrictive diet immediately. Answer: B Explanation: (A) She is probably not compliant with her diet and exercise program. Recommended weight gain during second and third trimesters is approximately 12 lb. (B) Because of her excessive weight gain of 10 lb in 2 months, she needs re-evaluation of her eating habits and reinforcement of proper dietary habits for pregnancy. A 2200-calorie diet is recommended for most pregnant women with a weight gain of 27–30 lb over the 9-month period. With rapid and excessive weightgain, PIH should also be suspected. (C) She does not need to increase her caloric intake, but she does need to re-evaluate dietary habits. Ten pounds in 2 months is excessive weight gain during pregnancy, and health teaching is warranted. (D) Restrictive dieting is not recommended during pregnancy. QUESTION 47 The priority nursing goal when working with an autistic child is: A. To establish trust with the child B. To maintain communication with the family C. To promote involvement in school activities D. To maintain nutritional requirements Answer: A Explanation: (A) The priority nursing goal when working with an autistic child is establishing a trusting relationship. (B) Maintaining a relationship with the family is important but having the trust of the child is a priority. (C) To promote involvement in school activities is inappropriate for a child who is autistic. (D) Maintaining nutritional requirements is not the primary problem of the autistic child. QUESTION 48 The nurse would need to monitor the serum glucose levels of a client receiving which of the following medications, owing to its effects on glycogenolysis and insulin release? A. Norepinephrine (Levophed) B. Dobutamine (Dobutrex) C. Propranolol (Inderal) D. Epinephrine (Adrenalin) Answer: D Explanation: (A) Norepinephrine’s side effects are primarily related to safe, effective care environment and include NCLEX NCLEX-RN and headache. (C) Propranolol’s side effects include elevated blood urea nitrogen, serum transaminase, alkaline phosphatase, and lactic dehydrogenase. (D) Epinephrine increases serum glucose levels by increasing glycogenolysis and inhibiting insulin release. Prolonged use can elevate serum lactate levels, leading to metabolic acidosis, increased urinary catecholamines, false elevation of blood urea nitrogen, and decreased coagulation time. QUESTION 49 When a client questions the nurse as to the purpose of exercise electrocardiography (ECG) in the diagnosis of cardiovascular disorders, the nurse’s response should be based on the fact that: A. The test provides a baseline for further tests B. The procedure simulates usual daily activity and myocardial performance C. The client can be monitored while cardiac conditioning and heart toning are done D. Ischemia can be diagnosed because exercise increases O2 consumption and demand Answer: D Explanation: (A) The purpose of the study is not to provide a baseline for further tests. (B) The test causes an increase in O2 demand beyond that required to perform usual daily activities. (C) Monitoring does occur, but the test is not for the purpose of cardiac toning and conditioning. (D) Exercise ECG, or stress testing, is designed to elevate the peripheral and myocardial needs for O2 to evaluate the ability of the myocardium and coronary arteries to meet the additional demands. QUESTION 50 The following medications were noted on review of the client’s home medication profile. Which of the medications would most likely potentiate or elevate serum digoxin levels? A. KCl B. Thyroid agents C. Quinidine D. Theophylline Answer: C Explanation: (A) Hypokalemia can cause digoxin toxicity. Administration of KCl would prevent this. (B) Thyroid agents decrease digoxin levels. (C) Quinidine increases digoxin levels dramatically. (D) Theophylline is not noted to have an effect on digoxin levels. QUESTION 51 Which of the following statements relevant to a suicidal client is correct? A. The more specific a client’s plan, the more likely he or she is to attempt suicide. B. A client who is unsuccessful at a first suicide attempt is not likely to make future attempts. C. A client who threatens suicide is just seeking attention and is not likely to attempt suicide. D. Nurses who care for a client who has attempted suicide should not make any reference to the word “suicide” in order to protect the client’s ego. NCLEX NCLEX-RN Explanation: (A, B, D) These occur in both tension pneumothorax and open pneumothorax. (C) The tension pneumothorax acts like a one- way valve so that the pneumothorax increases with each breath. Eventually, it occupies enough space to shift mediastinal tissue toward the unaffected side away from the midline. Tracheal deviation, movement of point of maximum impulse, and decreased cardiac output will occur. The other three options will occur in both types of pneumothorax. QUESTION 58 A 38-year-old pregnant woman visits her nurse practitioner for her regular prenatal checkup. She is 30 weeks’ gestation. The nurse should be alert to which condition related to her age? A. Iron-deficiency anemia B. Sexually transmitted disease (STD) C. Intrauterine growth retardation D. Pregnancy-induced hypertension (PIH) Answer: D Explanation: (A) Iron-deficiency anemia can occur throughout pregnancy and is not age related. (B) STDs can occur prior to or during pregnancy and are not age related. (C) Intrauterine growth retardation is an abnormal process where fetal development and maturation are delayed. It is not age related. (D) Physical risks for the pregnant client older than 35 include increased risk for PIH, cesarean delivery, fetal and neonatal mortality, and trisomy. QUESTION 59 A type I diabetic client is diagnosed with cellulitis in his right lower extremity. The nurse would expect which of the following to be present in relation to his blood sugar level? A. A normal blood sugar level B. A decreased blood sugar level C. An increased blood sugar level D. Fluctuating levels with a predawn increase Answer: C Explanation: (A) Blood sugar levels increase when the body responds to stress and illness. (B) Blood sugar levels increase when the body responds to stress and illness. (C) Hyperglycemia occurs because glucose is produced as the body responds to the stress and illness of cellulitis. (D) Blood sugar levels remain elevated as long as the body responds to stress and illness. QUESTION 60 A laboratory technique specific for diagnosing Lyme disease is: A. Polymerase chain reaction B. Heterophil antibody test C. Decreased serum calcium level D. Increased serum potassium level Answer: A NCLEX NCLEX-RN B. Exacerbation of depressive symptoms Explanation: (A) Polymerase chain reaction is the laboratory technique specific for Lyme disease. (B) Heterophil antibody test is used to diagnose mononucleosis. (C) Lyme disease does not decrease the serum calcium level. (D) Lyme disease does not increase the serum potassium level. QUESTION 61 The cardiac client who exhibits the symptoms of disorientation, lethargy, and seizures may be exhibiting a toxic reaction to: A. Digoxin (Lanoxin) B. Lidocaine (Xylocaine) C. Quinidine gluconate or sulfate (Quinaglute, Quinidex) D. Nitroglycerin IV (Tridil) Answer: B Explanation: (A) Side effects of digoxin include headache, hypotension, AV block, blurred vision, and yellow-green halos. (B) Side effects of lidocaine include heart block, headache, dizziness, confusion, tremor, lethargy, and convulsions. (C) Side effects of quinidine include heart block, hepatotoxicity, thrombocytopenia, and respiratory depression. (D) Side effects of nitroglycerin include postural hypotension, headache, dizziness, and flushing. QUESTION 62 A 27-year-old man was diagnosed with type I diabetes 3 months ago. Two weeks ago he complained of pain, redness, and tenderness in his right lower leg. He is admitted to the hospital with a slight elevation of temperature and vague complaints of “not feeling well.” At 4:30 PM on the day of his admission, his blood glucose level is 50 mg; dinner will be served at 5:00 PM. The best nursing action would be to: A. Give him 3 tbsp of sugar dissolved in 4 oz of grape juice to drink B. Ask him to dissolve three pieces of hard candy in his mouth C. Have him drink 4 oz of orange juice D. Monitor him closely until dinner arrives Answer: C Explanation: (A) The combination of sugar and juice will increase the blood sugar beyond the normal range. (B) Concentrated sweets are not absorbed as fast as juice; consequently, they elevate the blood sugar beyond the normal limit. (C) Four ounces of orange juice will act immediately to raise the blood sugar to a normal level and sustain it for 30 minutes until supper is served. (D) There is an increased potential for the client’s blood sugar to decrease even further, resulting in diabetic coma. QUESTION 63 After 3 weeks of treatment, a severely depressed client suddenly begins to feel better and starts interacting appropriately with other clients and staff. The nurse knows that this client has an increased risk for: A. Suicide NCLEX NCLEX-RN B. Epinephrine C. Violence toward others D. Psychotic behavior Answer: A Explanation: (A) When the severely depressed client suddenly begins to feel better, it often indicates that the client has made the decision to kill himself or herself and has developed a plan to do so. (B) Improvement in behavior is not indicative of an exacerbation of depressive symptoms. (C) Thedepressed client has a tendency for self-violence, not violence toward others. (D) Depressive behavior is not always accompanied by psychotic behavior. QUESTION 64 Discharge teaching was effective if the parents of a child with atopic dermatitis could state the importance of: A. Maintaining a high-humidified environment B. Furry, soft stuffed animals for play C. Showering 3–4 times a day D. Wrapping hands in soft cotton gloves Answer: D Explanation: (A) Maintaining a low-humidified environment. (B) Avoiding furry, soft stuffed animals for play, which may increase symptoms of allergy. (C) Avoiding showering, which irritates the dermatitis, and encouraging bathing 4 times a day in colloid bath for temporary relief. (D) Wrapping hands in soft cotton gloves to prevent skin damage during scratching. QUESTION 65 Which of the following would differentiate acute from chronic respiratory acidosis in the assessment of the trauma client? A. Increased PaCO2 B. Decreased PaO2 C. Increased HCO3 D. Decreased base excess Answer: C Explanation: (A) Increased CO2 will occur in both acute and chronic respiratory acidosis. (B) Hypoxia does not determine acid-base status. (C) Elevation of HCO3 is a compensatory mechanism in acidosis that occurs almost immediately, but it takes hours to show any effect and days to reach maximum compensation. Renal disease and diuretic therapy may impair the ability of the kidneys to compensate. (D) Base excess is a nonrespiratory contributor to acid-base balance. It would increase to compensate for acidosis. QUESTION 66 When a client is receiving vasoactive therapy IV, such as dopamine (Intropin), and extravasation occurs, the nurse should be prepared to administer which of the following medications directly into the site? A. Phentolamine (Regitine) NCLEX NCLEX-RN C. Ticks D. Mosquitoes Answer: C Explanation: (A) Mites are not the common vector of Lyme disease. (B) Fleas are not the common vector of Lyme disease. (C) Ticks are the common vector of Lyme disease. (D) Mosquitoes are not the common vector of Lyme disease. QUESTION 73 A child is admitted to the emergency room with her mother. Her mother states that she has been exposed to chickenpox. During the assessment, the nurse would note a characteristic rash: A. That is covered with vesicular scabs all in the macular stage B. That appears profusely on the trunk and sparsely on the extremities C. That first appears on the neck and spreads downward D. That appears especially on the cheeks, which gives a “slapped-cheek” appearance Answer: B Explanation: (A) A rash with vesicular scabs in all stages (macule, papule, vesicle, and crusts). (B) A rash that appears profusely on the trunk and sparsely on the extremities. (C) A rash that first appears on the neck and spreads downward is characteristic of rubeola and rubella. (D) A rash, especially on the cheeks, that gives a “slapped-cheek” appearance is characteristic of roseola. QUESTION 74 A client is 6 weeks pregnant. During her first prenatal visit, she asks, “How much alcohol is safe to drink during pregnancy?” The nurse’s response is: A. Up to 1 oz daily B. Up to 2 oz daily C. Up to 4 oz weekly D. No alcohol Answer: D Explanation: (A, B, C) No amount of alcohol has been determined safe for pregnant women. Alcohol should be avoided owing to the risk of fetal alcohol syndrome. (D) The recommended safe dosage of alcohol consumption during pregnancy is none. QUESTION 75 Which of the following would the nurse expect to find following respiratory assessment of a client with advanced emphysema? A. Distant breath sounds B. Increased heart sounds C. Decreased anteroposterior chest diameter D. Collapsed neck veins NCLEX NCLEX-RN Answer: A Explanation: (A) Distant breath sounds are found in clients with emphysema owing to increased anteroposterior chest diameter, overdistention, and air trapping. (B) Deceased heart sounds arepresent because of the increased anteroposterior chest diameter. (C) A barrel-shaped chest is characteristic of emphysem A. (D) Increased distention of neck veins is found owing to right-sided heart failure, which may be present in advanced emphysema. QUESTION 76 Which of the following would have the physiological effect of decreasing intracranial pressure (ICP)? A. Increased core body temperature B. Decreased serum osmolality C. Administration of hypo-osmolar fluids D. Decreased PaCO2 Answer: D Explanation: (A) An increase in core body temperature increases metabolism and results in an increase in ICP. (B) Decreased serum osmolality indicates a fluid overload and may result in an increase in ICP. (C) Hypo-osmolar fluids are generally voided in the neurologically compromised. Using IV fluids such as D5W results in the dextrose being metabolized, releasing free water that is absorbed by the brain cells, leading to cerebral edem A. (D) Hypercapnia and hypoventilation, which cause retention of CO2 and lead to respiratory acidosis, both increase ICP. CO2 is the most potent vasodilator known. QUESTION 77 When teaching a mother of a 4-month-old with diarrhea about the importance of preventing dehydration, the nurse would inform the mother about the importance of feeding her child: A. Fruit juices B. Diluted carbonated drinks C. Soy-based, lactose-free formula D. Regular formulas mixed with electrolyte solutions Answer: C Explanation: (A) Diluted fruit juices are not recommended for rehydration because they tend to aggravate the diarrhe A. (B) Diluted soft drinks have a high-carbohydrate content, which aggravates the diarrhe A. (C) Soy-based, lactose-free formula reduces stool output and duration of diarrhea in most infants. (D) Regular formulas contain lactose, which can increase diarrhea. QUESTION 78 The nurse is aware that nutrition is an important aspect of care for a client with hepatitis. Which of the following diets would be most therapeutic? A. High protein and low carbohydrate B. Low calorie and low protein NCLEX NCLEX-RN C. High carbohydrate and high calorie D. Low carbohydrate and high calorie Answer: C Explanation: (A) Protein increases the workload of the liver. Increased carbohydrates provide needed calories and promote palatability. (B) Dietary intake should be adequate to ensure wound healing. (C) Increased carbohydrates provide needed calories. (D) A highcalorie diet is best obtained from carbohydrates because of their palatability. Fats increase the workload of the liver. QUESTION 79 A long-term goal for the nurse in planning care for a depressed, suicidal client would be to: A. Provide him with a safe and structured environment. B. Assist him to develop more effective coping mechanisms. C. Have him sign a “no-suicide” contract. D. Isolate him from stressful situations that may precipitate a depressive episode. Answer: B Explanation: (A) This statement represents a short-term goal. (B) Long-term therapy should be directed toward assisting the client to cope effectively with stress. (C) Suicide contracts represent short-term interventions. (D) This statement represents an unrealistic goal. Stressful situations cannot be avoided in reality. QUESTION 80 The primary reason that an increase in heart rate (100 bpm) detrimental to the client with a myocardial infarction (MI) is that: A. Stroke volume and blood pressure will drop proportionately B. Systolic ejection time will decrease, thereby decreasing cardiac output C. Decreased contractile strength will occur due to decreased filling time D. Decreased coronary artery perfusion due to decreased diastolic filling time will occur, which will increase ischemic damage to the myocardium Answer: D Explanation: (A) Decreased stroke volume and blood pressure will occur secondary to decreased diastolic filling. (B) Tachycardia primarily decreases diastole; systolic time changes very little. (C) Contractility decreases owing to the decreased filling time and decreased time for fiber lengthening. (D) Decreased O2 supply due to decreased time for filling of the coronary arteriesincreases ischemia and infarct size. Tachycardia primarily robs the heart of diastolic time, which is the primary time for coronary artery filling. QUESTION 81 Which of the following nursing orders should be included in the plan of care for a client with hepatitis C? A. The nurse should use universal precautions when obtaining blood samples. B. Total bed rest should be maintained until the client is asymptomatic. C. The client should be instructed to maintain a low semi-Fowler position when eating meals. NCLEX NCLEX-RN QUESTION 87 Which of the following procedures is necessary to establish a definitive diagnosis of breast cancer? A. Diaphanography B. Mammography C. Thermography D. Breast tissue biopsy Answer: D Explanation: (A) Diaphanography, also known as transillumination, is a painless, noninvasive imaging technique that involves shining a light source through the breast tissue to visualize the interior. It must be used in conjunction with a mammogram and physical examination. (B) Mammography is a useful tool for screening but is not considered a means of diagnosing breast cancers. (C) Thermography is a pictorial representation of heat patterns on the surface of the breast. Breast cancers appear as a “hot spot” owing to their higher metabolic rate. (D) Biopsy either by needle aspiration or by surgical incision is the primary diagnostic technique for confirming the presence of cancer cells. QUESTION 88 Pregnant women with diabetes often have problems related to the effectiveness of insulin in controlling their glucose levels during their second half of pregnancy. The nurse teaches the client that this is due to: A. Decreased glomerular filtration and increased tubular absorption B. Decreased estrogen levels C. Decreased progesterone levels D. Increased human placental lactogen levels Answer: D Explanation: (A) There is a rise in glomerular filtration rate in the kidneys in conjunction with decreased tubular glucose reabsorption, resulting in glycosuri A. (B) Insulin is inhibited by increased levels of estrogen. (C) Insulin is inhibited by increased levels of progesterone. (D) Human placental lactogen levels increase later in pregnancy. This hormonal antagonist reduces insulin’s effectiveness, stimulates lipolysis, and increases the circulation of free fatty acids. QUESTION 89 A pregnant woman at 36 weeks’ gestation is followed for PIH and develops proteinuri A. To increase protein in her diet, which of the following foods will provide the greatest amount of protein when added to her intake of 100 mL of milk? A. Fifty milliliters light cream and 2 tbsp corn syrup B. Thirty grams powdered skim milk and 1 egg C. One small scoop (90 g) vanilla ice cream and 1 tbsp chocolate syrup D. One package vitamin-fortified gelatin drink Answer: B Explanation: NCLEX NCLEX-RN (A) This choice would provide more unwanted fat and sugar than protein. (B) Skim milk would add protein. Eggs are good sources of protein while low in fat and calories. (C) The benefit of protein from ice cream would be outweighed by the fat content. Chocolate syrup has caffeine, which is contraindicated or limited in pregnancy. (D) Although most animal proteins are higher in protein than plant proteins, gelatin is not. It loses protein during the processing for food consumption. QUESTION 90 A 3-year-old child is hospitalized with burns covering her trunk and lower extremities. Which of the following would the nurse use to assess adequacy of fluid resuscitation in the burned child? A. Blood pressure B. Serum potassium level C. Urine output D. Pulse rate Answer: C Explanation: (A) Blood pressure can remain normotensive even in a state of hypovolemi A. (B) Serum potassium is not reliable for determining adequacy of fluid resuscitation. (C) Urine output, alteration in sensorium, and capillary refill are the most reliable indicators for assessing adequacy of fluid resuscitation. (D) Pulse rate may vary for many reasons and is not a reliable indicator for assessing adequacy of fluid resuscitation. QUESTION 91 The medication that best penetrates eschar is: A. Mafenide acetate (Sulfamylon) B. Silver sulfadiazine (Silvadene) C. Neomycin sulfate (Neosporin) D. Povidone-iodine (Betadine) Answer: A Explanation: (A) Mafenide acetate is bacteriostatic against gram-positive and gram-negative organisms and is the agent that best penetrates eschar. (B) Silver sulfadiazine poorly penetrates eschar. (C) Neomycin sulfate does not penetrate eschar. (D) Povidoneiodine does not penetrate eschar. QUESTION 92 A 25-year-old client believes she may be pregnant with her first child. She schedules an obstetric examination with the nurse practitioner to determine the status of her possible pregnancy. Her last menstrual period began May 20, and her estimated date of confinement using Nägele’s rule is: A. March 27 B. February 1 C. February 27 D. January 3 NCLEX NCLEX-RN Answer: C Explanation: (A)March 27 is a miscalculation. (B) February 1 is a miscalculation. (C) February 27 is the correct answer. To calculate the estimated date of confinement using Nagele’s rule, subtract 3 months from the date that the last menstrual cycle began and then add 7 days to the result. (D) January 3 is a miscalculation. QUESTION 93 The therapeutic blood-level range for lithium is: A. 0.25–1.0 mEq/L B. 0.5–1.5 mEq/L C. 1.0–2.0 mEq/L D. 2.0–2.5 mEq/L Answer: B Explanation: (A) This range is too low to be therapeutic. (B) This is the therapeutic range for lithium. (C) This range is above the therapeutic level. (D) This range is toxic and may cause severe side effects. QUESTION 94 Which of the following ECG changes would be seen as a positive myocardial stress test response? A. Hyperacute T wave B. Prolongation of the PR interval C. ST-segment depression D. Pathological Q wave Answer: C Explanation: (A) Hyperacute T waves occur with hyperkalemi A. (B) Prolongation of the P R interval occurs with first-degree AV block. (C) Horizontal ST-segment depression of>1 mm during exercise isdefinitely a positive criterion on the exercise ECG test. (D) Patho-logical Q waves occur with MI. QUESTION 95 A psychotic client who believes that he is God and rules all the universe is experiencing which type of delusion? A. Somatic B. Grandiose C. Persecutory D. Nihilistic Answer: B Explanation: (A) These delusions are related to the belief that an individual has an incurable illness. (B) These delusions are related to feelings of self-importance and uniqueness. (C) These delusions are related to feelings of being conspired against. (D) These delusions are related to denial of self-existence. NCLEX NCLEX-RN D. 2-1-1-0-2 Answer: C Explanation: (A) This answer is an incorrect application of the GTPAL method. One prior pregnancy was a preterm birth at 36 weeks (T =1, P= 1; not T = 2). (B) This answer is an incorrect application of the GTPAL method. The client is currently pregnant for the third time (G = 3, not 2), one prior pregnancy was preterm (T= 1, P= 1; not T= 2), and she has had no prior abortions (A =0). (C) This answer is the correct application of GTPAL method. The client is currently pregnant for the third time (G =3), her first pregnancy ended at term (>37 weeks) (T = 1), her second pregnancy ended preterm 20–33 weeks) (P = 1), she has no history of abortion (A=0), and she has two living children (L = 2). (D) This answer is an incorrect application of the GTPAL method. The client is currently pregnant for the third time (G =3, not 2). QUESTION 102 A 68-year-old woman is admitted to the hospital with chronic obstructive pulmonary disease (COPD). She is started on an aminophylline infusion. Three days later she is breathing easier. A serum theophylline level is drawn. Which of the following values represents a therapeutic level? A. 14 µ g/mL B. 25 µ g/mL C. 4 µ g/mL D. 30 µ g/mL Answer: A Explanation: (A) The therapeutic blood level range of theophylline is 10–20 mg/mL. Therapeutic drug monitoring determines effective drug dosages and prevents toxicity. (B, D) This value is a toxic level of the drug. (C) This value is a nontherapeutic level of the drug. QUESTION 103 A nasogastric (NG) tube inserted preoperatively is attached to low, intermittent suctions. A client with an NG tube exhibits these symptoms: He is restless; serum electrolytes are Na 138, K 4.0, blood pH 7.53. This client is most likely experiencing: A. Hyperkalemia B. Hyponatremia C. Metabolic acidosis D. Metabolic alkalosis Answer: D Explanation: (A) Sodium level is within normal limits. (B) Sodium level is within normal limits. (C) pH level is consistent with alkalosis. (D) With an NG tube attached to low, intermittent suction, acids are removed and a client will develop metabolic alkalosis. QUESTION 104 A client is in early labor. Her fetus is in a left occipitoanterior (LOA) position; fetal heart sounds are best NCLEX NCLEX-RN auscultated just: A. Below the umbilicus toward left side of mother’s abdomen B. Below the umbilicus toward right side of mother’s abdomen C. At the umbilicus D. Above the umbilicus to the left side of mother’s abdomen Answer: A Explanation: (A) LOA identifies a fetus whose back is on its mother’s left side, whose head is the presenting part, and whose back is toward its mother’s anterior. It is easiest to auscultate fetal heart tones (FHTs) through the fetus’s back. (B) The identified fetus’s back is on its mother’s left side, not right side. It is easiest to auscultate FHTs through the fetus’s back. (C) In an LOA position, the fetus’s head is presenting with the back to the left anterior side of the mother. The umbilicus is too high of a landmark for auscultating the fetus’s heart rate through its back. (D) This is the correct auscultation point for a fetus in the left sacroanterior position, where the sacrum is presenting, not LOA. QUESTION 105 A mother came to the pediatric clinic with her 17- month-old child. The mother would like to begin toilet training. What should the nurse teach her about implementing toilet training? A. Take two or three favorite toys with the child. B. Have a child-sized toilet seat or training potty on hand. C. Explain to the child she is going to “void” and “defecate.” D. Show disapproval if she does not void or defecate. Answer: B Explanation: (A) Giving her toys will distract her and interfere with toilet training because of inappropriate reinforcement. (B) A child-sized toilet seat or training potty gives a child a feeling of security. (C) She should use words that are age appropriate for the child. (D) Children should be praised for cooperative behavior and/or successful evacuation. QUESTION 106 Which one of the following is considered a reliable indicator for assessing the adequacy of fluid resuscitation in a 3-year-old child who suffered partial- and fullthickness burns to 25% of her body? A. Urine output B. Edema C. Hypertension D. Bulging fontanelle Answer: A Explanation: (A) Urinary output is a reliable indicator of renal perfusion, which in turn indicates that fluid resuscitation is adequate. IV fluids are adjusted based on the urinary output of the child during fluid resuscitation. (B) Edema is an indication of increased capillary permeability following a burn injury. (C) Hypertension is an indicator of fluid volume excess. (D) Fontanelles close by 18 months of age. NCLEX NCLEX-RN QUESTION 107 The nurse working in a prenatal clinic needs to be alert to the cardinal signs and symptoms of PIH because: A. Immediate treatment of mild PIH includes the administration of a variety of medications B. Psychological counseling is indicated to reduce the emotional stress causing the blood pressure elevation C. Self-discipline is required to control caloric intake throughout the pregnancy D. The client may not recognize the early symptoms of PIH Answer: D Explanation: (A) Mild PIH is not treated with medications. (B) Emotional stress is not the cause of blood pressure elevation in PIH. (C) Excessive caloric intake is not the cause of weight gain in PIH. (D) The client most frequently is not aware of the signs and symptoms in mild PIH. QUESTION 108 A client is admitted to the labor unit. On vaginal examination, the presenting part in a cephalic presentation was at station plus two. Station 12 means that the: A. Presenting part is 2 cm above the level of the ischial spines B. Biparietal diameter is at the level of the ischial spines C. Presenting part is 2 cm below the level of the ischial spines D. Biparietal diameter is 5 cm above the ischial spines Answer: C Explanation: (A) Station is the relationship of the presenting part to an imaginary line drawn between the ischial spines. If the presenting part is above the ischial spines, the station is negative. (B) When the biparietal diameter is at the level of the ischial spines, the presenting part is generally at a +4 or +5 station. (C) Station is the relationship of the presenting part to an imaginary line drawn between the ischial spines. If the presenting part is below the ischial spines, the station is positive. Thus, 2 cm below the ischial spines is the station +2. (D) When the biparietal diameter is above the ischial spines by 5 cm, the presenting part is usually engaged or at station 0. QUESTION 109 To prevent fungal infections of the mouth and throat, the nurse should teach clients on inhaled steroids to: A. Rinse the plastic holder that aerosolizes the drug with hydrogen peroxide every other day B. Rinse the mouth and gargle with warm water after each use of the inhaler C. Take antacids immediately before inhalation to neutralize mucous membranes and prevent infection D. Rinse the mouth before each use to eliminate colonization of bacteria Answer: B Explanation: (A) It is sufficient to rinse the plastic holders with warm water at least once per day. (B) It is important to rinse the mouth after each use to minimize the risk of fungal infections by reducing the droplets of the glucocorticoid left in the oral cavity. (C) Antacids act by neutralizing or reducing gastric acid, thus decreasing the pH of the stomach. “Neutralizing” the oral mucosa prior to inhalation of a steroid inhaler does not minimize the risk of fungal NCLEX NCLEX-RN A. High Fowler B. Prone C. Supine D. Right lateral Answer: D Explanation: (A) This position does not help to prevent bleeding. (B) This position does not help to prevent bleeding. (C) This position does not help to prevent bleeding. (D) The right lateral position would allow pressure on the liver to prevent bleeding. QUESTION 116 In assisting preconceptual clients, the nurse should teach that the corpus luteum secretes progesterone, which thickens the endometrial lining in which of the phases of the menstrual cycle? A. Menstrual phase B. Proliferative phase C. Secretory phase D. Ischemic phase Answer: C Explanation: (A) Menses occurs during the menstrual phase, during which levels of both estrogen and progesterone are decreased. (B) The ovarian hormone responsible for the proliferative phase, during which the uterine endometrium enlarges, is estrogen. (C) The ovarian hormone responsible for the secretory phase is progesterone, which is secreted by the corpus luteum and causes marked swelling in the uterine endometrium. (D) The corpus luteum begins to degenerate in the ischemic phase, causing a fall in both estrogen and progesterone. QUESTION 117 A 4-year-old child is being discharged from the hospital after being treated for severe croup. Which one of the following instructions should the nurse give to the child’s mother for the home treatment of croup? A. Take him in the bathroom, turn on the hot water, and close the door. B. Give him a dose of antihistamine. C. Give large amounts of clear liquids if drooling occurs. D. Place him near a cool mist vaporizer and encourage crying. Answer: A Explanation: (A) Initial home treatment of croup includes placing the child in an environment of high humidity to liquefy and mobilize secretions. (B) Antihistamines should be avoided because they can cause thickening of secretions. (C) Drooling is a characteristic sign of airway obstruction and the child should be taken directly to the emergency room. (D) Crying increases respiratory distress and hypoxia in the child with croup. The nurse should promote methods that will calm the child. QUESTION 118 A male client is started on IV anticoagulant therapy with heparin. Which of the following laboratory studies will be ordered to monitor the therapeutic effects of heparin? NCLEX NCLEX-RN A. Partial thromboplastin time B. Hemoglobin C. Red blood cell (RBC) count D. Prothrombin time Answer: A Explanation: (A) Partial thromboplastin time is used to monitor the effects of heparin, and dosage is adjusted depending on test results. It is a screening test used to detect deficiencies in all plasma clotting factors except factors VII and XIII and platelets. (B) Hemoglobin is the main component of RBCs. Its main function is to carry O2from the lungs to the body tissues and to transport CO2back to the lungs. (C) RBC count is the determination of the number of RBCs found in each cubic millimeter of whole blood. (D) PT is used to monitor the effects of oral anticoagulants, e.g., coumarintype anticoagulants. QUESTION 119 A 16-year-old student has a long history of bronchial asthma and has experienced several severe asthmatic attacks during the school year. The school nurse is required to administer 0.2 mL of 1/1000 solution of epinephrine SC during an asthma attack. How does the school nurse evaluate the effectiveness of this intervention? A. Increased pulse rate B. Increased expectorate of secretions C. Decreased inspiratory difficulty D. Increased respiratory rate Answer: C Explanation: (A) A side effect of epinephrine is fatal ventricular fibrillation owing to its effects on cardiac stimulation. (B) Medications used to treat asthma are designed to decrease bronchospasm, not to increase expectorate of secretions. (C) Epinephrine decreased inspiratory difficulty by stimulating -, 1, and 2-receptors causing sympathomimetic stimulation (e.g., bronchodilation). (D) The person with asthma fights to inspire sufficient air thus increasing respiratory rate. QUESTION 120 Nursing interventions designed to decrease the risk of infection in a client with an indwelling catheter include: A. Cleanse area around the meatus twice a day B. Empty the catheter drainage bag at least daily C. Change the catheter tubing and bag every 48 hours D. Maintain fluid intake of 1200–1500 mL every day Answer: A Explanation: (A) Catheter site care is to be done at least twice daily to prevent pathogen growth at the catheter insertion site. (B) Catheter drainage bags are usually emptied every 8 hours to prevent urine stasis and pathogen growth. (C) Tubing and collection bags are not changed this often, because research studies have not demonstrated the efficacy of this practice. NCLEX NCLEX-RN (D) Fluid intake needs to be in the 2000–2500 mL range if possible to help irrigate the bladder and prevent infection. QUESTION 121 A 23-year-old borderline client is admitted to an inpatient psychiatric unit following an impulsive act of self- mutilation. A few hours after admission, she requests special privileges, and when these are not granted, she stands up and angrily shouts that the people on the unit do not care, and she storms across the room. The nurse should respond to this behavior by: A. Placing her in seclusion until the behavior is under control B. Walking up to the client and touching her on the arm to get her attention C. Communicating a desire to assist the client to regain control, offering a one-to-one session in a quiet area D. Confronting the client, letting her know the consequences for getting angry and disrupting the unit Answer: C Explanation: (A) Threatening a client with punitive action is violating a client’s rights and could escalate the client’s anger. (B) Angry clients need respect for personal space, and physical contact may be perceived as a threatening gesture escalating anger. (C) Client lacks sufficient self-control to limit own maladaptive behavior; she may need assistance from staff. (D) Confronting an angry client may escalate her anger to further acting out, and consequences are for acting out anger aggressively, not for getting angry or feeling angry. QUESTION 122 A client is being discharged with albuterol (Proventil) and beclomethasone dipropionate (Vanceril) to be administered via inhalation three times a day and at bedtime. Client teaching regarding the sequential order in which the drugs should be administered includes: A. Glucocorticoid followed by the bronchodilator B. Bronchodilator followed by the glucocorticoid C. Alternate successive administrations D. According to the client’s preference Answer: B Explanation: (A) The client would not receive therapeutic effects of the glucocorticoid when it is inhaled through constricted airways. (B) Bronchodilating the airways first allows for the glucocorticoid to be inhaled through open airways and increases the penetration of the steroid for maximum effectiveness of the drug. (C) Inac- Inaccurate use of the inhalers will lead to decreased effectiveness of the treatment. (D) Client teaching regarding the use and effects of inhalers will promote client understanding and compliance. QUESTION 123 A gravida 2 para 1 client delivered a full-term newborn 12 hours ago. The nurse finds her uterus to be boggy, high, and deviated to the right. The most appropriate nursing action is to: A. Notify the physician B. Place the client on a pad count C. Massage the uterus and re-evaluate in 30 minutes D. Have the client void and then re-evaluate the fundus NCLEX NCLEX-RN QUESTION 129 A client is being discharged and will continue enteral feedings at home. Which of the following statements by a family member indicates the need for further teaching? A. “If he develops diarrhea lasting for more than 2–3 days, I will contact the doctor or nurse.” B. “I should anticipate that he will gain about 1 lb/day now that he is on continuous feedings.” C. “It is important to keep the head of his bed elevated or sit him in the chair during feedings.” D. “I should use prepared or open formula within 24 hours and store unused portions in the refrigerator.” Answer: B Explanation: (A) Diarrhea is a complication of tube feedings that can lead to dehydration. Diarrhea may be the result of hypertonic formulas that can draw fluid into the bowel. Other causes of diarrhea may be bacterial contamination, fecal impaction, medications, and low albumin. (B) A consistent weight gain of more than 0.22 kg/day (12 lb/day) over several days should be reported promptly. The client should be evaluated for fluid volume excess. (C) Elevating the client’s head prevents reflux and thus formula from entering the airway. (D) Bacteria proliferate rapidly in enteral formulas and can cause gastroenteritis and even sepsis. QUESTION 130 Which nursing implication is appropriate for a client undergoing a paracentesis? A. Have the client void before the procedure. B. Keep the client NPO. C. Observe the client for hypertension following the procedure. D. Place the client on the right side following the procedure. Answer: A Explanation: (A) A full bladder would impede withdrawal of ascitic fluid. (B) Keeping the client NPO is not necessary. (C) The client may exhibit signs and symptoms of shock and hypertension. (D) No position change is needed after the procedure. QUESTION 131 One of the medications that is prescribed for a male client is furosemide (Lasix) 80 mg bid. To reduce his risk of falls, the nurse would teach him to take this medication: A. On arising and no later than 6 PM B. At evenly spaced intervals, such as 8 AM and 8 PM C. With at least one glass of water per pill D. With breakfast and at bedtime Answer: A Explanation: (A) This option provides adequate spacing of the medication and will limit the client’s need to get up to go to the bathroom during the night hours, when he is especially at high risk for falls. (B) This option would result in the need to get up during the night to urinate and would thus increase the risk of falls. This option also does not take into consideration the client’s usual daily routine. (C) Taking this medication with at least one glass of NCLEX NCLEX-RN water would not have an impact on the risk of falls. (D) This option would result in the need to get up during the night to urinate and would thus increase the risk of falls. QUESTION 132 The nurse observes that a client has difficulty chewing and swallowing her food. A nursing response designed to reduce this problem would include: A. Ordering a full liquid diet for her B. Ordering five small meals for her C. Ordering a mechanical soft diet for her D. Ordering a puréed diet for her Answer: C Explanation: (A) Full liquids would be difficult to swallow if the muscle control of the swallowing act is affected; this is a probable reason for her difficulties, given her medical diagnosis of multiple sclerosis. (B) Five small meals would do little if anything to decrease her swallowing difficulties, other than assure that she tires less easily. (C) A mechanical soft diet should be easier to chew and swallow, because foods would be more evenly consistent. (D) A pureed diet would cause her to regress more than might be needed; the mechanical soft diet should be tried first. QUESTION 133 In performing the initial nursing assessment on a client at the prenatal clinic, the nurse will know that which of the following alterations is abnormal during pregnancy? A. Striae gravidarum B. Chloasma C. Dysuria D. Colostrum Answer: C Explanation: (A) Striae gravidarum are the normal stretch marks that frequently occur on the breasts, abdomen, and thighs as pregnancy progresses. (B) Chloasma is the “mask of pregnancy” that normally occurs in many pregnant women. (C) Dysuria is an abnormal danger sign during pregnancy and may indicate a urinary tract infection. (D) Colostrum is a yellow breast secretion that is normally present during the last trimester of pregnancy. QUESTION 134 Assessment of severe depression in a client reveals feelings of hopelessness, worthlessness; inability to feel pleasure; sleep, psychomotor, and nutritional alterations; delusional thinking; negative view of self; and feelings of abandonment. These clinical features of the client’s depression alert the nurse to prioritize problems and care by addressing which of the following problems first: A. Nutritional status B. Impaired thinking C. Possible harm to self D. Rest and activity impairment NCLEX NCLEX-RN Answer: C Explanation: (A) Anorexia and weight loss are problems that need attention in severe depression, but they can be addressed secondary to immediate concerns. (B) Impaired thinking and confusion are problems in severe depression that are addressed with administration of medication, through group and individual psychotherapy, and through activity therapy as motivation and interest increase. (C) Possible harm to self as with suicidal ideation; a suicide plan, means to execute plan; and/or overt gestures or an attempt must be addressed as an immediate concern and safety measures implemented appropriate to the risk of suicide. (D) Rest and activity impairment may take time and further assessment to determine client’s sleep pattern and amount of psychomotor retardation with the more immediate concern for safety present. QUESTION 135 The nurse is teaching a mother care of her child’s spica cast. The mother states that he complains of itching under the edge of the cast. One nonpharmacological technique the nurse might suggest would be: A. “Blowing air under the cast using a hair dryer on cool setting often relieves itching.” B. “Slide a ruler under the cast and scratch the area.” C. “Guide a towel under and through the cast and move it back and forth to relieve the itch.” D. “Gently thump on cast to dislodge dried skin that causes the itching.” Answer: A Explanation: (A) Cool air will often relieve pruritus without damaging the cast or irritating the skin. (B) The nurse should never force anything under the cast, because the cast may become damaged and skin breakdown may occur. (C) Forcing an object under the cast could lead to cast damage and skin breakdown. The object may become lodged under the cast necessitating cast removal. (D) This technique does not dislodge skin cells. It could damage the cast and cause skin breakdown. QUESTION 136 A 56-year-old psychiatric inpatient has had recurring episodes of depression and chronic low self-esteem. She feels that her family does not want her around, experiences a sense of helplessness, and has a negative view of herself. To assist the client in focusing on her strengths and positive traits, a strategy used by the nurse would be to: A. Tell the client to attend all structured activities on the unit B. Encourage or direct client to attend activities that offer simple methods to attain success C. Increase the client’s self-esteem by asking that she make all decisions regarding attendance in group activities D. Not allow any dependent behaviors by the client because she must learn independence and will have to ask for any assistance from staff Answer: B Explanation: (A) The nurse should encourage activities gradually, as client’s energy level and tolerance for shared activities improve. (B) Activities that focus on strengths and accomplishments, with uncomplicated tasks, minimize failure and increase self-worth. (C) Asking a client to set a goal to make all decisions about attending group activities is unrealistic, and such decisions are not always under the client’s control; this sets up the client for NCLEX NCLEX-RN A. Is also called intrinsic factor B. Must be given in the abdomen C. Requires use of the Z-track method D. Should be given SC Answer: C Explanation: (A) Intrinsic factor is needed to absorb vitamin B12.(B) Iron dextran is given parenterally, but Z-track in a large muscle. (C) A Ztrack method of injection is required to prevent staining and irritation of the tissue. (D) An SC injection is not deep enough and may cause subcutaneous fat abscess formation. QUESTION 143 The nurse would be sure to instruct a client on the signs and symptoms of an eye infection and hemorrhage. These signs and symptoms would include: A. Blurred vision and dizziness B. Eye pain and itching C. Feeling of eye pressure and headache D. Eye discharge and hemoptysis Answer: B Explanation: (A) Although blurred vision may occur, dizziness would not be associated with an infection or hemorrhage. (B) Eye pain is a symptom of hemorrhage within the eye, and itching is associated with infection. (C) Nausea and headache would not be usual symptoms of eye hemorrhage or infection. (D) Some eye discharge might be anticipated if an infection is present; hemoptysis would not. QUESTION 144 The nurse is caring for a laboring client. Assessment data include cervical dilation 9 cm; contractions every 1–2 minutes; strong, large amount of “bloody show.” The most appropriate nursing goal for this client would be: A. Maintain client’s privacy. B. Assist with assessment procedures. C. Provide strategies to maintain client control. D. Enlist additional caregiver support to ensure client’s safety. Answer: C Explanation: (A) Privacy may help the laboring client feel safer, but measures that enhance coping take priority. (B) The frequency of assessments do increase in transition, but helping the client to maintain control and cope with this phase of labor takes on importance. (C) This laboring client is in transition, the most difficult part of the first stage of labor because of decreased frequency, increased duration and intensity, and decreased resting phase of the uterine contraction. The client’s ability to cope is most threatened during this phase of labor, and nursing actions aredirected toward helping the client to maintain control. (D) Safety is a concern throughout labor, but helping the client to cope takes on importance in transition. QUESTION 145 A complication for which the nurse should be alert following a liver biopsy is: NCLEX NCLEX-RN A. Hepatic coma B. Jaundice C. Ascites D. Shock Answer: D Explanation: (A) Hepatic coma may occur in liver disease due to the increased NH3levels, not due to liver biopsy. (B) Jaundice may occur due to increased bilirubin levels, not due to liver biopsy. (C) Ascites would occur due to portal hypertension, not due to liver biopsy. (D) Hemorrhage and shock are the most likely complications after liver biopsy because of already existing bleeding tendencies in the vascular makeup of the liver. QUESTION 146 A 42-year-old client on an inpatient psychiatric unit comments that he was brought to the hospital by his wife because he had taken too many pills and states, “I just couldn’t take it anymore.” The nurse’s best response to this disclosure would be: A. “You shouldn’t do things like that, just tell someone you feel bad.” B. “Tell me more about what you couldn’t take anymore.” C. “I’m sure you probably didn’t mean to kill yourself.” D. “How long have you been in the hospital.” Answer: B Explanation: (A) Disapproving gives the impression that the nurse has a right to pass judgment on the client’s thoughts, actions, or ideas. (B) Giving a broad opening gives the client encouragement to continue with verbalization. (C) Failing to acknowledge the client’s feelings conveys a lack of understanding and empathy. (D) Changing the subject takes the conversation away from the client and is indicative of the nurse’s anxiety or insensitivity. QUESTION 147 After a 10-year-old child with insulin-dependent diabetes mellitus receives her dinner tray, she tells the nurse that she hates broccoli and wants some corn on the cob. The nurse’s appropriate response is: A. “No vegetable exchanges are allowed.” B. “Corn and other starchy vegetables are considered to be bread exchanges.” C. “Yes, you may exchange any vegetable for any other vegetable.” D. “Yes, but only one-half ear is allowed.” Answer: B Explanation: (A) Sites for injection need to be rotated, including abdominal sites, to enhance insulin absorption. (B) The pinch technique is the most effective method for obtaining skin tightness to allow easy entrance of the needle to subcutaneous tissues. (C) Massaging the site of injectionfacilitates absorption of the insulin. (D) Changing the needle will break the sterility of the system. It has become acceptable practice to reuse disposable needles and syringes for 3–7 days. NCLEX NCLEX-RN QUESTION 148 Which of the following signs might indicate a complication during the labor process with vertex presentation? A. Fetal tachycardia to 170 bpm during a contraction B. Nausea and vomiting at 8–10 cm dilation C. Contraction lasting 60 seconds D. Appearance of dark-colored amniotic fluid Answer: D Explanation: (A) Fetal tachycardia may indicate fetal hypoxia; however, 170 bpm is only mild tachycardi A. (B) Nausea and vomiting occur frequently during transition and are not a complication. (C) Contractions frequently last 60–90 seconds during the transitional phase of labor and are not considered a complication as long as the uterus relaxes completely between contractions. (D) Passage of meconium in a vertex presentation is a sign of fetal distress; this may be normal in a breech presentation owing to pressure on the presenting part. QUESTION 149 The nurse would be concerned if a client exhibited which of the following symptoms during her postpartum stay? A. Pulse rate of 50–70 bpm by her third postpartum day B. Diuresis by her second or third postpartum day C. Vaginal discharge or rubra, serosa, then rubra D. Diaphoresis by her third postpartum day Answer: C Explanation: (A) Bradycardia is an expected assessment during the postpartum period. (B) Diuresis can occur during labor and the postpartum period and is an expected physiological adaptation. (C) A return of rubra after the serosa period may indicate a postpartal complication. (D) Diaphoresis, especially at night, is an expected physiological change and does not indicate an infectious process. Bradycardia, diuresis, and diaphoresis are normal postpartum physiological responses to adjust the cardiac output and blood volume to the nonpregnant state. QUESTION 150 MgSO4 blood levels are monitored and the nurse would be prepared to administer the following antidote for MgSO4 side effects or toxicity: A. Magnesium oxide B. Calcium hydroxide C. Calcium gluconate D. Naloxone (Narcan) Answer: C Explanation: (A, B) These drugs are not antidotes for MgSO4. (C) This drug is the standard antidote and should always be readily available when MgSO4is being administered. (D) This drug is an antidote for narcotics, not MgSO4. NCLEX NCLEX-RN A. 130/88 to 144/92 B. 136/90 to 148/100 C. 150/96 to 160/104 D. 118/70 to 130/88 Answer: D Explanation: (A, B, C) The individual’s systolic and diastolic changes are more significant than the relatively high initial blood pressure readings. (D) The systolic pressure went up 12 mm Hg and the diastolic pressure 18 mm Hg. This is a more significant rise than the increases in A–C choices, and client should receive more frequent evaluations and care. QUESTION 157 The nurse would assess the client’s correct understanding of the fertility awareness methods that enhance conception, if the client stated that: A. “My sexual partner and I should have intercourse when my cervical mucosa is thick and cloudy.” B. “At ovulation, my basal body temperature should rise about 0.5F.” C. “I should douche immediately after intercourse.” D. “My sexual partner and I should have sexual intercourse on day 14 of my cycle regardless of the length of the cycle.” Answer: B Explanation: (A) At ovulation, the cervical mucus is increased, stretchable, and watery clear. (B) Under the influence of progesterone, the basal body temperature increases slightly after ovulation. (C) To enhance fertility, measures should be taken that promote retention of sperm rather than removal. (D) Ovulation, the optimal time for conception, occurs 14+2 days before the next menses; therefore, the date of ovulation is directly related to the length of the menstrual cycle. QUESTION 158 In evaluating the laboratory results of a client with severe pressure ulcers, the nurse finds that her albumin level is low. A decrease in serum albumin would contribute to the formation of pressure ulcers because: A. The proteins needed for tissue repair are diminished. B. The iron stores needed for tissue repair are inadequate. C. A decreased serum albumin level indicates kidney disease. D. A decreased serum albumin causes fluid movement into the blood vessels, causing dehydration. Answer: A Explanation: (A) Serum albumin levels indicate the adequacy of protein stores available for tissue repair. (B) Serum albumin does not measure iron stores. (C) Serum albumin levels do not measure kidney function. (D) A decreased serum albumin level would cause fluid movement out of blood vessels, not into them. NCLEX NCLEX-RN QUESTION 159 A client is being discharged on warfarin (Coumadin), an oral anticoagulant. The nurse instructs him about using this drug. Which following response by the client indicates the need for further teaching? A. “I should shave with my electric razor while on Coumadin.” B. “I will inform my dentist that I am on anticoagulant therapy before receiving dental work.” C. “I will continue with my usual dosage of aspirin for my arthritis when I return home.” D. “I will wear an ID bracelet stating that I am on anticoagulants.” Answer: C Explanation: (A) Using an electric razor prevents the risk of cuts while shaving. (B) Any physician or dentist should be informed of anticoagulant therapy because of the risk of bleeding due to a prolonged PT. (C) The client should be instructed to consult with his physician. Aspirin is avoided because it potentiates the affects of oral anticoagulants by interfering with platelet aggregation. (D) Identification bracelets are necessary to direct treatment, especially in an emergency situation. QUESTION 160 When a client with pancreatitis is discharged, the nurse needs to teach him how to prevent another occurrence of acute pancreatitis. Which of the following statements would indicate he has an understanding of his disease? A. “I will not eat any raw or uncooked vegetables.” B. “I will limit my alcohol to one cocktail per day.” C. “I will look into attending Alcoholics Anonymous meetings.” D. “I will report any changes in bowel movements to my doctor.” Answer: C Explanation: (A) Raw or uncooked vegetables are all right to eat postdischarge. (B) This client must avoid any alcohol intake. (C) The client displays awareness of the need to avoid alcohol. (D) This action would be pertinent only if fatty stools associated with chronic hepatitis were the problem. QUESTION 161 A client decided early in her pregnancy to breast-feed her first baby. She gave birth to a normal, full-term girl and is now progressing toward the establishment of successful lactation. To remove the baby from her breast, she should be instructed to: A. Gently pull the infant away B. Withdraw the breast from the infant’s mouth C. Compress the areolar tissue until the infant drops the nipple from her mouth D. Insert a clean finger into the baby’s mouth beside the nipple Answer: D Explanation: (A) In pulling the infant away from the breast without breaking suction, nipple trauma is likely to occur. (B) In pulling the breast away from the infant without breaking suction, nipple trauma is likely to occur. (C) Compressing the maternal tissue does not break the suction of the infant on the breast and can cause nipple traum NCLEX NCLEX-RN A. (D) By inserting a finger into the infant’s mouth beside the nipple, the lactating mother can break the suction and the nipple can be removed without trauma. QUESTION 162 A parent told the public health nurse that her 6-year-old son has been taking tetracycline for a chronic skin condition. The parent asked if this could cause any problems for the child. What should the nurse explain to the parent? A. Giving tetracycline to a child younger than 8 years may cause permanent staining of his teeth. B. If you give tetracycline with milk, it may be absorbed readily. C. The medication should be given to adults, not children. D. Secondary infections of chronic skin disorders do not respond to antibiotics. Answer: A Explanation: (A) Tetracycline should be avoided during tooth development because it interferes with enamel formation and dental pigmentation. (B) Milk interferes with the absorption of tetracyclines. (C) Children older than 9 years or past the tooth development stage may be given tetracycline. (D) Secondary infections of chronic skin disorders may respond to antibiotics such as penicillin or tetracyclines. QUESTION 163 MgSO4 is ordered IV following the established protocol for a client with severe PIH. The anticipated effects of this therapy are anticonvulsant and: A. Vasoconstrictive B. Vasodilative C. Hypertensive D. Antiemetic Answer: B Explanation: (A) An anticonvulsant effect is the goal of drug therapy for PIH. However, we would not want to increase the vasoconstriction that is already present. This would make the symptoms more severe. (B) An anticon-vulsant effect and vasodilation are the desired outcomes when administering this drug. (C) An anticonvulsant effect is the goal of drug therapy for PIH; however, hypertensive drugs would increase the blood pressure even more. (D) An anticonvulsant effect is the goal of drug therapy for PIH. MgSO4is not classified as an antiemetic. Antiemetics are not indicated for PIH treatment. QUESTION 164 The nurse is preparing a 6-year-old child for an IV insertion. Which one of the following statements by the nurse is appropriate when preparing a child for a potentially painful procedure? A. “Some say this feels like a pinch or a bug bite. You tell me what it feels like.” B. “This is going to hurt a lot; close your eyes and hold my hand.” C. “This is a terrible procedure, so don’t look.” D. “This will hurt only a little; try to be a big boy.” NCLEX NCLEX-RN very high and it’s best to avoid long discussions. (C) Manic clients have a tendency to manipulate persons in their environment. Staff should monitor intake, including at mealtime and snacks, and be consistent in their approach to meeting nutritional needs. (D) Manic clients may not sit and eat complete meals, but they can carry foods and liquids from regular meals with them. Staff can monitor and give high-caloric and high-energy snacks and liquids. QUESTION 170 A 35-weeks-pregnant client is undergoing a nonstress test (NST). During the 20-minute examination, the nurse notes three fetal movements accompanied by accelerations of the fetal heart rate, each 15 bpm, lasting 15 seconds. The nurse interprets this test to be: A. Nonreactive B. Reactive C. Positive D. Negative Answer: B Explanation: (A) In a nonreactive NST, the criteria for reactivity are not met. (B) A reactive NST shows at least two accelerations of FHR with fetal movements, each 15 bpm, lasting 15 seconds or more, over 20 minutes. (C, D) This term is used to interpret a contraction stress test (CST), or oxytocin challenge test, not an NST. QUESTION 171 A 30-year-old client has just been treated in the ER for bruises and abrasions to her face and a broken arm from domestic violence, which has been increasing in frequency and intensity over the last few months. The nurse assesses her as being very anxious, fearful, bewildered, and feeling helpless as she states, “I don’t know what to do, I’m afraid to go home.” The best response by the nurse to the client would be: A. “I wouldn’t want to go home either; call a friend who could help you.” B. “Did you do something that could have made him so angry?” C. “Let’s talk about people and resources available to you so that you don’t have to go home.” D. “I’ll call the police and they will take care of him, and you can go home and get some rest.” Answer: C Explanation: (A) A person in crisis needs support, assistance, and direction from a caregiver rather than just an instruction. (B) A battered person may feel guilt and think that they cause the abuser’s behavior; however, the abuser has the problem and goes through phases of violence. (C) The nurse should provide support and guidance to the client in crisis by offering alternatives and assist in referrals. (D) Focusing on help from law enforcement may be a very temporary solution, because the victim may be fearful of pressing charges. This answer does not address the crisis of going home. QUESTION 172 A 74-year-old obese man who has undergone open reduction and internal fixation of the right hip is 8 days postoperative. He has a history of arthritis and atrial fibrillation. He admits to right lower leg pain, described as “a cramp in my leg.” An appropriate nursing action is to: NCLEX NCLEX-RN A. Assess for pain with plantiflexion B. Assess for edema and heat of the right leg C. Instruct him to rub the cramp out of his leg D. Elevate right lower extremity with pillows propped under the knee Answer: B Explanation: (A) Calf pain with dorsiflexion of the foot (Homans’ sign) can be a sign of a deep venous thrombosis; however, it is not diagnostic of the condition. (B) Swelling and warmth along the affected vein are commonly observed clinical manifestations of a deep venous thrombosis as a result of inflammation of the vessel wall. (C) Rubbing or massaging of the affected leg is contraindicated because of the risk of the clot breaking loose and becoming an embolus. (D) A pillow behind the knee can be constricting and further impair blood flow. QUESTION 173 A 26-year-old client is admitted to the labor, delivery, recovery, postpartum unit. The nurse completes her assessment and determines the client is in the first stage of labor. The nurse should instruct her: A. To hold her breath during contractions B. To be flat on her back C. Not to push with her contractions D. To push before becoming fully dilated Answer: C Explanation: (A) This nursing action may cause hyperventilation. (B) This nursing action could cause inferior vena cava syndrome. (C) The client is allowed to push only after complete dilation during the second stage of labor. The nurse needs to know the stages of labor. (D) If the client pushes before dilation, it could cause cervical edema and/or edema to the fetal scalp; both of these could contribute to increased risk of complications. QUESTION 174 A client in active labor asks the nurse for coaching with her breathing during contractions. The client has attended Lamaze birth preparation classes. Which of the following is the best response by the nurse? A. “Keep breathing with your abdominal muscles as long as you can.” B. “Make sure you take a deep cleansing breath as the contractions start, focus on an object, and breathe about 16–20 times a minute with shallow chest breaths.” C. “Find a comfortable position before you start a contraction. Once the contraction has started, take slow breaths using your abdominal muscles.” D. “If a woman in labor listens to her body and takes rapid, deep breaths, she will be able to deal with her contractions quite well.” Answer: B Explanation: (A) Lamaze childbirth preparation teaches the use of chest, not abdominal, breathing. (B) In Lamaze preparation, every patterned breath is preceded by a cleansing breath; as labor progresses, shallow, paced breathing is found to be effective. (C) It is important to assume a comfortable position in labor, but the NCLEX NCLEX-RN Lamazeprepared laboring woman is taught to breathe with her chest, not abdominal, muscles. (D) When deep chest breathing patterns are used in Lamaze preparation, they are slowly paced at a rate of 6–9 breaths/min. QUESTION 175 A couple is planning the conception of their first child. The wife, whose normal menstrual cycle is 34 days in length, correctly identifies the time that she is most likely to ovulate if she states that ovulation should occur on day: A. 14+2 days B. 16+2 days C. 20+2 days D. 22+2 days Answer: C Explanation: (A) Ovulation is dependent on average length of menstrual cycle, not standard 14 days. (B) Ovulation occurs 14+2 days before next menses (34 minus 14 does not equal 16). (C) Ovulation occurs 14+2 days before next menses (34 minus 14 equals 20). (D) Ovulation occurs 14+2 days before next menses (34 minus 14 does not equal 22). QUESTION 176 A nurse should carefully monitor a client for the following side effect of MgSO4: A. Visual blurring B. Tachypnea C. Epigastric pain D. Respiratory depression Answer: D Explanation: (A, C) The nurse should provide good distractors because these symptoms indicate that PIH has become more severe and may precede the convulsive or eclamptic phase. (B) This is the oppositeside effect of this medication. (D) This is a common side effect of this medication and needs to be monitored and recorded frequently. QUESTION 177 The nurse would teach a male client ways to minimize the risk of infection after eye surgery. Which of the following indicates the client needs further teaching? A. “I will wash my hands before instilling eye medications.” B. “I will wear sunglasses when going outside.” C. “I will wear an eye patch for the first 3 postoperative days.” D. “I will maintain the sterility of the eye medications.” Answer: C Explanation: (A) Hand washing would be an important action designed to prevent transmission of pathogens from the hands to the eye. (B) Wearing sunglasses when going outside will prevent airborne NCLEX NCLEX-RN C. Avoid applying pressure after injection. D. Change needles after injection. Answer: B Explanation: (A) Sites for injection need to be rotated, including abdominal sites, to enhance insulin absorption. (B) The pinch technique is the most effective method for obtaining skin tightness to allow easy entrance of the needle to subcutaneous tissues. (C) Massaging the site of injection facilitates absorption of the insulin. (D) Changing the needle will break the sterility of the system. It has become acceptable practice to reuse disposable needles and syringes for 3–7 days. QUESTION 184 A client is experiencing muscle weakness and lethargy. His serum K+is 3.2. What other symptoms might he exhibit? A. Tetany B. Dysrhythmias C. Numbness of extremities D. Headache Answer: B Explanation: (A) Tetany is seen with low calcium. (B) Low potassium causes dysrhythmias because potassium is responsible for cardiac muscle activity. (C) Numbness of extremities is seen with high potassium. (D) Headache is not associated with potassium excess or deficiency. QUESTION 185 Other drugs may be ordered to manage a client’s ulcerative colitis. Which of the following medications, if ordered, would the nurse question? A. Methylprednisolone sodium succinate (Solu-Medrol) B. Loperamide (Imodium) C. Psyllium D. 6-Mercaptopurine Answer: D Explanation: (A) Methylprednisolone sodium succinate is used for its anti-inflammatory effects. (B) Loperamide would be used to control diarrhe A. (C) Psyllium may improve consistency of stools by providing bulk. (D) An immunosuppressant such as 6- mercaptopurine is used for chronic unrelenting Crohn’s disease. QUESTION 186 An 8-week-old infant has been diagnosed with gastroesophageal reflux. The nurse is teaching the infant’s mother to care for the infant at home. Which one of the following statements by the nurse is appropriate regarding the infant’s home care? A. “Lay the infant flat on her left side after feeding.” NCLEX NCLEX-RN B. “Feed the infant every 4 hours with half-strength formula.” C. “Antacids need to be given an hour before feeding.” D. “Play activities should be carried out before instead of after feedings.” Answer: D Explanation: (A) Elevating the child’s head to a 30-degree angle is the recommended position for gastroesophageal reflux. The supine position predisposes the child to aspiration. (B) Small, frequent feedings with thickened formula are recommended to minimize vomiting. (C) Antacids should be given at the same time as the feeding to improve their buffering action. (D) The infant should be kept still after feedings to reduce the risk of vomiting and aspiration. Vigorous activities should be carried out before feedings. QUESTION 187 A mother is unsure about the type of toys for her 17-month-old child. Based on knowledge of growth and development, what toy would the nurse suggest? A. A pull toy to encourage locomotion B. A mobile to improve hand-eye coordination C. A large toy with movable parts to improve pincer grasp D. Various large colored blocks to teach visual discrimination Answer: A Explanation: (A) Increased locomotive skills make push-pull toys appropriate for the energetic toddler. (B) Infants progress from reflex activity through simple repetitive behaviors to imitative behavior. Hand-eye coordination forms the foundation of other movements. (C) At age 8 months, infants begin to have pincer grasp. Toys that help infants develop the pincer grasp are recommended for this age group. (D) Various large colored blocks are suggested toys for infants 6–12 months of age to help visual stimulation. QUESTION 188 A group of nursing students at a local preschool day care center are going to screen each child’s fine and gross motor, language, and social skills. The students will use which one of the most widely used screening tests? A. Revised Prescreening Developmental Questionnaire B. Goodenough Draw-a-Person Screening Test C. Denver Development Screening Test D. Caldwell Home Inventory Answer: C Explanation: (A) The Revised Prescreening Developmental Questionnaire is more age appropriate and offers simplified parent scoring and easier comparison. It is used by parents instead of professionals. (B) The Goodenough Draw- a-Person test is used to assess intellectual development. (C) The Denver Developmental Screening Test is one of the most widely used screening tests. It offers a concise, easy-to-administer, systematic approach to assessing the preschool child. It is widely used because of its reliability and validity. (D) The Caldwell Home Inventory is used to assess the home environment in areas of social, emotional, and cognitive supports. NCLEX NCLEX-RN QUESTION 189 A gravida 2 para 1 client is hospitalized with severe preeclampsi A. While she receives magnesium sulfate (MgSO4) therapy, the nurse knows it is safe to repeat the dosage if: A. Deep tendon reflexes are absent B. Urine output is 20 mL/hr C. MgSO4serum levels are>15 mg/dL D. Respirations are>16 breaths/min Answer: D Explanation: (A) MgSO4is a central nervous system depressant. Loss of reflexes is often the first sign of developing toxicity. (B) Urinary output at <25 mL/hr or 100 mL in 4 hours may result in the accumulation of toxic levels of magnesium. (C) The therapeutic serum range for MgSO4is 6–8 mg/dL. Higher levels indicate toxicity. (D) Respirations of>16 breaths/min indicate that toxic levels of magnesium have not been reached. Medication administration would be safe. QUESTION 190 In addition to changing the mother’s position to relieve cord pressure, the nurse may employ the following measure (s) in the event that she observes the cord out of the vagina: A. Immediately pour sterile saline on the cord, and repeat this every 15 minutes to prevent drying. B. Cover the cord with a wet sponge. C. Apply a cord clamp to the exposed cord, and cover with a sterile towel. D. Keep the cord warm and moist by continuous applications of warm, sterile saline compresses. Answer: D Explanation: (A) Saline should be warmed; waiting 15 minutes may not keep the cord moist. (B) This choice does not specify what the sponge was “wet” with. (C) This measure would stop circulation to the fetus. (D) The cord should be kept warm and moist to maintain fetal circulation. This measure is an accepted nursing action. QUESTION 191 Following a gastric resection, which of the following actions would the nurse reinforce with the client in order to alleviate the distress from dumping syndrome? A. Eating three large meals a day B. Drinking small amounts of liquids with meals C. Taking a long walk after meals D. Eating a low-carbohydrate diet Answer: D Explanation: (A) Six small meals are recommended. (B) Liquids after meals increase the time food empties from the stomach. (C) Lying down after meals is recommended to prevent gravity from producing dumping. (D) A low- carbohydrate diet will prevent a hypertonic bolus, which causes dumping. NCLEX NCLEX-RN food choices indicate that this teaching has been understood? A. Omelette and hash browns B. Pancakes and syrup C. Bagel with cream cheese D. Cooked oatmeal and grapefruit half Answer: D Explanation: (A) Eggs and hash browns do not provide much fiber and bulk, so they do not effectively prevent constipation. (B) Pancakes and syrup also have little fiber and bulk, so they do not effectively prevent constipation. (C) Bagel and cream cheese do not provide intestinal bulk. (D) A combination of oatmeal and fresh fruit will provide fiber and intestinal bulk. QUESTION 198 In cleansing the perineal area around the site of catheter insertion, the nurse would: A. Wipe the catheter toward the urinary meatus B. Wipe the catheter away from the urinary meatus C. Apply a small amount of talcum powder after drying the perineal area D. Gently insert the catheter another 12 inch after cleansing to prevent irritation from the balloon Answer: B Explanation: (A) Wiping toward the urinary meatus would transport microorganisms from the external tubing to the urethra, thereby increasing the risk of bladder infection. (B) Wiping away from the urinary meatus would remove microorganisms from the point of insertion of the catheter, thereby decreasing the risk of bladder infection. (C) Talcum powder should not be applied following catheter care, because powders contribute to moisture retention and infection likelihood. (D) The catheter should never be inserted further into the urethra, because this would serve no useful purpose and would increase the risk of infection. QUESTION 199 A 7-year-old child is brought to the ER at midnight by his mother after symptoms appeared abruptly. The nurse’s initial assessment reveals a temperature of 104.5F (40.3C), difficulty swallowing, drooling, absence of a spontaneous cough, and agitation. These symptoms are indicative of which one of the following? A. Acute tracheitis B. Acute spasmodic croup C. Acute epiglottis D. Acute laryngotracheobronchitis Answer: C Explanation: (A) Clinical manifestations of acute tracheitis include a 2–3 day history of URI, croupy cough, stridor, purulent secretions, high fever. (B) Clinical manifestations of spasmodic croup include a history of URI, croupy cough, stridor, dyspnea, low-grade fever, and a slow progression. The age group most affected is 3 months to 3 years. (C) Three clinical observations have been found to be predictive of epiglottitis: the presence of drooling, absence of spontaneous cough, and agitation.Epiglottitis has a rapid onset that is accompanied by high fever and NCLEX NCLEX-RN dysphagi A. (D) Clinical manifestations of acute laryngotracheobronchitis (LTB) include slow onset with a history of URI, low-grade fever, stridor, brassy cough, and irritability. QUESTION 200 A client tells the nurse that she has had a history of urinary tract infections. The nurse would do further health teaching if she verbalizes she will: A. Drink at least 8 oz of cranberry juice daily B. Maintain a fluid intake of at least 2000 mL daily C. Wash her hands before and after voiding D. Limit her fluid intake after 6 PM so that there is not a great deal of urine in her bladder while she sleeps Answer: D Explanation: (A) Cranberry juice helps to maintain urine acidity, thereby retarding bacterial growth. (B) A generous fluid intake will help to irrigate the bladder and to prevent bacterial growth within the bladder. (C) Hand washing is an effective means of preventing pathogen transmission. (D) Restricting fluid intake would contribute to urinary stasis, which in turn would contribute to bacterial growth. QUESTION 201 A pregnant client experiences a precipitous delivery. The nursing action during a precipitous delivery is to: A. Control the delivery by guiding expulsion of fetus B. Leave the room to call the physician C. Push against the perineum to stop delivery D. Cross client’s legs tightly Answer: A Explanation: (A) Controlling the rapid delivery will reduce the risk of fetal injury and perineal lacerations. (B) The nurse should always remain with a client experiencing a precipitous delivery. (C) Pushing against the perineum may cause fetal distress. (D) Crossing of legs may cause fetal distress and does not stop the delivery process. QUESTION 202 A client is diagnosed with organic brain disorder. The nursing care should include: A. Organized, safe environment B. Long, extended family visits C. Detailed explanations of procedures D. Challenging educational programs Answer: A Explanation: (A) A priority nursing goal is attending to the client’s safety and well-being. Reorient frequently, remove dangerous objects, and maintain consistent environment. (B) Short, NCLEX NCLEX-RN frequent visits are recommended to avoid overstimulation and fatigue. (C) Short, concise, simple explanations are easier to understand. (D) Mental capability and attention span deficits make learning difficult and frustrating. QUESTION 203 A premature infant needs supplemental O2 therapy. A nursing intervention that reduces the risk of retrolental fibroplasia is to: A. Maintain O2at <40% B. Maintain O2at>40% C. Give moist O2at>40% D. Maintain on 100% O2 Answer: A Explanation: (A) Retrolental fibroplasia is the result of prolonged exposure to high levels of O2in premature infants. Complications are hemorrhage and retinal detachment. (B, C, D) O2concentration is too high. QUESTION 204 A 35-year-old client is admitted to the hospital with diabetic ketoacidosis. Results of arterial blood gases are pH 7.2, PaO2 90, PaCO2 45, and HCO3 16. The nursing assessment of arterial blood gases indicate the presence of: A. Respiratory alkalosis B. Respiratory acidosis C. Metabolic alkalosis D. Metabolic acidosis Answer: D Explanation: (A) Respiratory alkalosis is determined by elevated pH and low PaCO2. (B) Respiratory acidosis is determined by low pH and elevated PaCO2. (C) Metabolic alkalosis is determined by elevated pH and HCO3.(D) Metabolic acidosis is determined by low pH and HCO3. QUESTION 205 A client presents to the psychiatric unit crying hysterically. She is diagnosed with severe anxiety disorder. The first nursing action is to: A. Demand that she relax B. Ask what is the problem C. Stand or sit next to her D. Give her something to do Answer: C Explanation: (A) This nursing action is too controlling and authoritative. It could increase the client’s anxiety level. (B) In her anxiety state, the client cannot rationally identify a problem. (C) This nursing action conveys a message of caring and security. (D) Giving the client a task would increase her anxiety. This would be a late nursing action. NCLEX NCLEX-RN because it: A. Prevents administration of other drugs B. Prevents entry of air into tubing C. Prevents inadvertent administration of a large amount of fluids D. Prevents phlebitis Answer: C Explanation: (A) A volume control set has a chamber that permits the administration of compatible drugs. (B) Air may enter a volume control set when tubing is not adequately purged. (C) A volume control set allows the nurse to control the amount of fluid administered over a set period. (D) Contamination of volume control set may cause phlebitis. QUESTION 212 A pregnant client experiences spontaneous rupture of membranes. The first nursing action is to: A. Assess the client’s respirations B. Notify the physician C. Auscultate fetal heart rate D. Transfer to delivery suite Answer: C Explanation: (A) Immediately following membrane rupture, the fetus is at risk for complications, not necessarily the mother. (B) The physician is notified after the nurse completes an assessment of the mother’s and fetus’s conditions. (C) Rupture of membranes facilitates fetal descent. A potential complication is cord prolapse, which is assessed by auscultating fetal heart rate. (D) Rupture of membranes does not necessarily indicate readiness to deliver. QUESTION 213 A 30-year-old client has a history of several recent traumatic experiences. She presents at the physician’s office with a complaint of blindness. Physical exam and diagnostic testing reveal no organic cause. The nurse recognizes this as: A. Delusion B. Illusion C. Hallucination D. Conversion Answer: D Explanation: (A) The client’s blindness is real. Delusion is a false belief. (B) Illusion is the misrepresentation of a real, external sensory experience. (C) Hallucination is a false sensory perception involving any of the senses. (D) Conversion is the expression of intrapsychic conflict through sensory or motor manifestations. QUESTION 214 A client was not using his seat belt when involved in a car accident. He fractured ribs 5, 6, and 7 on the left and developed a left pneumothorax. Assessment findings include: NCLEX NCLEX-RN A. Crackles and paradoxical chest wall movement B. Decreased breath sounds on the left and chest pain with movement C. Rhonchi and frothy sputum D. Wheezing and dry cough Answer: B Explanation: (A) Crackles are caused by air moving through moisture in the small airways and occur with pulmonary edem A. Paradoxical chest wall movement occurs with flail chest when a segment of the thorax moves outward on inspiration and inward on expiration. (B) Decreased breath sounds occur when a lung is collapsed or partially collapsed. Chest pain with movement occurs with rib fractures. (C) Rhonchi are caused by air moving through large fluid-filled airways. Frothy sputum may occur with pulmonary edem A. (D) Wheezing is caused by fluid in large airways already narrowed by mucus or bronchospasm. Dry cough could indicate a cardiac problem. QUESTION 215 An 11-year-old boy has received a partial-thickness burn to both legs. He presents to the emergency room approximately 15 minutes after the accident in excruciating pain with charred clothing to both legs. What is the first nursing action? A. Apply ice packs to both legs. B. Begin débridement by removing all charred clothing from wound. C. Apply Silvadene cream (silver sulfadiazine). D. Immerse both legs in cool water. Answer: D Explanation: (A) Ice creates a dramatic temperature change in the tissue, which can cause further thermal injury. (B) Charred clothing should not be removed from wound first. This creates further tissue damage. Débridement is not the first nursing action. (C) Applying silver sulfadiazine cream first insulates heat in injured tissue and increases potential for infection. (D) Emergency care of a thermal burn is immersing both legs in cool water. Cool water permits gradual temperature change and prevents further thermal damage. QUESTION 216 The physician is preparing to induce labor on a 40-week multigravid A. The nurse should anticipate the administration of: A. Oxytocin (Pitocin) B. Progesterone C. Vasopressin (Pitressin) D. Ergonovine maleate Answer: A Explanation: (A) Oxytocin is a hormone secreted by the neurohypophysis during suckling and parturition that produces NCLEX NCLEX-RN strong uterine contractions. (B) Progesterone has a quiescence effect on the uterus. (C) Vasopressin is an antidiuretic hormone that promotes water reabsorption by the renal tubules. (D) Ergonovine produces dystocia as a result of sustained uterine contractions. QUESTION 217 A pregnant client during labor is irritable and feels the urge to vomit. The nurse should recognize this as the: A. Fourth stage of labor B. Third stage of labor C. Transition stage of labor D. Second stage of labor Answer: C Explanation: (A) The fourth stage begins after expulsion of the placent A. Client symptoms are: fatigue; chills; scant, bloody vaginal discharge; and nause A. (B) The third stage is from birth to expulsion of placent A. Client symptoms are uterine contractions, gush of blood, and perineal pain. (C) The transition stage is characterized by strong uterine contractions and cervical dilation. Clientsymptoms are irritability, restlessness, belching, muscle tremors, nausea, and vomiting. (D) The second stage is characterized by full dilation of cervix. Client symptoms are perineal bulge, pushing with contractions, great irritability, and leg cramps. QUESTION 218 A physician’s order reads: Administer KCl 10% oral solution 1.5 mL. The KCl bottle reads 20 mEq/15 mL. What dosage should the nurse administer to the infant? A. 1 mEq B. 1.13 mEq C. 2 mEq D. Not enough information to calculate Answer: C Explanation: (A) This answer is a miscalculation. (B) This answer is a miscalculation. (C) 1.33 mEq = 1 mL, then 1.5 mL X=1.99, or 2 mEq. (D) Information is adequate for calculation. QUESTION 219 At 38 weeks’ gestation, a client is in active labor. She is using her Lamaze breathing techniques. The RN is coaching her breathing and encouraging her to relax and work with her contractions. Which one of the following complaints by the client will alert the RN that she is beginning to hyperventilate with her breathing? A. “I am cold.” B. “I have a backache.” C. “I feel dizzy.” D. “I am nauseous.” Answer: C Explanation: NCLEX NCLEX-RN separates from the uterine wall as the uterus contracts and cervix dilates. This separation causes painless bleeding in the 7th-8th month. (D) Polyhydramnios is excessive amniotic fluid. QUESTION 225 A 66-year-old female client has smoked 2 packs of cigarettes per day for 20 years. Her arterial blood gases on room air are as follows: pH 7.35; PO2 70 mm Hg; PCO2 55 mm Hg; HCO3 32 mEq/L. These blood gases reflect: A. Compensated metabolic acidosis B. Compensated respiratory acidosis C. Compensated respiratory alkalosis D. Uncompensated respiratory acidosis Answer: B Explanation: (A) In compensated metabolic acidosis, the pH level is normal, the PCO2level is decreased, and the HCO3level is decreased. The client’s primary alteration is an inability to remove excess acid via the kidneys. The lungs compensate by hyperventilating and decreasing PCO2. (B) In compensated respiratory acidosis, the pH level is normal, the PCO2level is elevated, and the HCO3level is elevated. The client’s primary alteration is an inability to remove CO2from the lungs, so over time, the kidneys increase reabsorption of HCO3to buffer the CO2. (C) In compensated respiratory alkalosis, the pH level is normal, the PCO2level is decreased, and the HCO3level is decreased. The client’s primary alteration is hyperventilation, which decreases PCO2. The client compensates by increasing the excretion of HCO3from the body. (D) In uncompensated respiratory acidosis, the pH level is decreased, the PCO2level is increased, and the HCO3level is normal. The client’s primary alteration is an inability to remove CO2from the lungs. The kidneys have not compensated by increasing HCO3reabsorption. QUESTION 226 The client has been in active labor for the last 12 hours. During the last 3 hours, labor has been augmented with oxytocin because of hypoactive uterine contractions. Her physician assesses her cervix as 95% effaced, 8 cm dilated, and the fetus is at 0 station. Her oral temperature is 100.2F at this time. The physician orders that she be prepared for a cesarean delivery. In preparing the client for the cesarean delivery, which one of the following physician’s orders should the RN question? A. Administer meperidine (Demerol) 100 mg IM 1 hour prior to the delivery. B. Discontinue the oxytocin infusion. C. Insert an indwelling Foley catheter prior to delivery. D. Prepare abdominal area from below the nipples to below the symphysis pubis are A. Answer: A Explanation: (A) Meperidine is a narcotic analgesic medication that crosses the placental barrier and reaches the fetus, causing respiratory depression in the fetus. A narcotic medication should never be included in the preoperative order for a cesarean delivery. (B) Oxytocin infusion would be discontinued if client is being prepared for a cesarean delivery because the medication would not be needed. (C) The bladder is always emptied prior to and during the surgical intervention to prevent the urinary bladder from accidentally being incised while the uterine incision is made. (D) The abdominal area is always prepared to rid the area of hair before the abdominal NCLEX NCLEX-RN incision is made. Abdominal hair cannot be sterilized and could become a source for postoperative incisional infection. QUESTION 227 A 17-year-old client has a T-4 spinal cord injury. At present, he is learning to catheterize himself. When he says, “This is too much trouble. I would rather just have a Foley.’’ An appropriate response for the RN teaching him would be: A. “I know. It is a lot to learn. In the long run, though, you will be able to reduce infections if you do an intermittent catheterization program.’’ B. “It is not too much trouble. This is the best way to manage urination.’’ C. “OK. I’ll ask your physician if we can replace the Foley.’’ D. “You need to learn this because your doctor ordered it.’’ Answer: A Explanation: (A) This response acknowledges the client’s feelings, gives him factual information, and acknowledges that the final decision is his. (B) This response is judgmental and discourages the client from expressing his feelings about the procedure. (C) Catheterization is a procedure thattakes time to learn, but which, for the spinal cord– injured client, can significantly reduce the incidence of urinary tract infections. A young client with a T-4 injury has the hand function to learn this procedure fairly easily. (D) The final decision about bladder elimination management ultimately rests with the client and not the physician. QUESTION 228 A cardinal symptom of the schizophrenic client is hallucinations. A nurse identifies this as a problem in the category of: A. Impaired communication B. Sensory-perceptual alterations C. Altered thought processes D. Impaired social interaction Answer: B Explanation: (A) Impaired communication refers to decreased ability or inability to use or understand language in an interaction. (B) In sensory-perceptual alterations an individual has distorted, impaired, or exaggerated responses to incoming stimuli (i.e., a hallucination, which is a false sensory perception that is not associated with real external stimuli). (C) An altered thought processes problem statement is used when an individual experiences a disruption in cognitive operations and activities (i.e., delusions, loose associations, ideas of reference). (D) In impaired social interaction, the individual participates too little or too much in social interactions. QUESTION 229 A client calls the prenatal clinic to schedule an appointment. She states she has missed three menstrual periods and thinks she might be pregnant. During her first visit to the prenatal clinic, it is confirmed that she is pregnant. The registered nurse (RN) learns that her last menstrual period began on June 10. According to Nägele’s rule, the estimated date of confinement is: NCLEX NCLEX-RN A. March 17 B. June 3 C. August 30 D. January 10 Answer: A Explanation: (A) Using Nägele’s rule, count back 3 calendar months from the first day of the last menstrual period. The answer is March 10. Then add 7 days and 1 year, which would be March 17 of the following year. (B, C, D) This date is incorrect. QUESTION 230 A 55-year-old client is unconscious, and his physician has decided to begin tube feeding him using a smallbore silicone feeding tube (Keofeed, Duo-Tube). After the tube is inserted, the nurse identifies the most reliable way to confirm appropriate placement is to: A. Aspirate gastric contents B. Auscultate air insufflated through the tube C. Obtain a chest x-ray D. Place the tip of the tube under water and observe for air bubbles Answer: C Explanation: (A) Aspiration of gastric contents is usually a reliable way to verify tube placement. However, if the client has dark respiratory secretions from bleeding, tube feedings could be mistaken for respiratory secretions; in other words, aspirating an empty stomach is less reliable in this instance. In addition, it is common for small-bore feeding tubes to collapse when suction pressure is applied. (B) Insufflation of air into large-bore nasogastric tubes can usually be clearly heard. In small-bore tubes, it is more difficult to hear air, and it is difficult to distinguish between air in the stomach and air in the esophagus. (C) A chest x-ray is the most reliable means to determine placement of small-bore nasogastric tubes. (D) Observing for air bubbles when the tip is held under water is an unreliable means to determine correct tube placement for all types of nasogastric tubes. Air may come from both the respiratory tract and the stomach, and the client who is breathing shallowly may not force air out of the tube into the water. QUESTION 231 A client had a vaginal delivery 3 days ago and is discharged from the hospital on the 2nd day postpartum. She told the RN, “I need to start exercising so that I can get back into shape. Could you suggest an exercise I could begin with?’’ The RN could suggest which one of the following? A. Push-ups B. Jumping jacks C. Leg lifts D. Kegel exercises Answer: D Explanation: (A, B, C) This exercise is too strenuous at this time. (D) This exercise is recommended for the first few days
Docsity logo



Copyright © 2024 Ladybird Srl - Via Leonardo da Vinci 16, 10126, Torino, Italy - VAT 10816460017 - All rights reserved